Adult Health Midterm

Ace your homework & exams now with Quizwiz!

Place the following events that occur during healing by primary intention in sequential order from 1 (first) to 10 (last). A. Blood clots form B. Avascular, pale, mature scar present C. Accumulation of inflammatory debris D. Enzymes from neutrophils digest fibrin E. Epithelial cells migrate across would surface F. Fibroblasts migrate to site and secrete collagen G. Budding capillaries result in pink, vascular friable would H. Contraction of healing area by movement of myofibroblasts I. Macrophages ingest and digest cellular debris and red blood cells J. Fibrin clot that sees as meshwork for capillary growth and epithelial cell migration

1- A. Blood clots form 10- B. Avascular, pale, mature scar present 2- C. Accumulation of inflammatory debris 4- D. Enzymes from neutrophils digest fibrin 8- E. Epithelial cells migrate across would surface 6- F. Fibroblasts migrate to site and secrete collagen 7- G. Budding capillaries result in pink, vascular friable would 9- H. Contraction of healing area by movement of myofibroblasts 3- I. Macrophages ingest and digest cellular debris and red blood cells 5- J. Fibrin clot that sees as meshwork for capillary growth and epithelial cell migration

The three antibiotic resistant bacteria that are the most current concern for North America

1. MERSA 2. VRE 3. PRSP

A patient with consistent dietary intake who loses 1 kg of weight in 1 day has lost _______

1000 mL of fluid 1 kg wt. = 1,000 mL fluid

The nurse notes drainage on the surgical dressing when the patient is transferred from the PACU to the clinical unit. in what order of priority should the nurse do the following actions? Number the options with 1 for the first action a 5 to the last action. A. Reinforce surgical dressing B. Change the dressing and access the wound as ordered C. Notify the surgeon of excessive drainage type of drain in use D. Recall the report from PACU for the number and type of drains in use. E. Note and record the type, amount, and color and odor of the drainage.

2 - A. Reinforce surgical dressing 5 - B. Change the dressing and access the wound as ordered 4 - C. Notify the surgeon of excessive drainage type of drain in use 1 - D. Recall the report from PACU for the number and type of drains in use. 3 - E. Note and record the type, amount, and color and odor of the drainage.

Match the methods of local anesthetic administration with their descriptions. A. Injection of agent into subarachnoid space B. Injection of anesthetic agent directly into tissues C. Injection of specific nerve with anesthetic agent D. Injection of anesthetic agent into space around vertebrae E. Injection of agent into veins of extremity after limb us exsanguinated

3 - A. Injection of agent into subarachnoid space 5 - B. Injection of anesthetic agent directly into tissues 1 - C. Injection of specific nerve with anesthetic agent 4 - D. Injection of anesthetic agent into space around vertebrae 2 - E. Injection of agent into veins of extremity after limb us exsanguinated

Number in sequence from 1-6 the pathophysiologic processes that occur in osteoarthritis (OA). A. Erosion of articular surfaces B. Incongruity in joint surfaces C. Reduction in motion D. Joint cartilage becomes yellow and granular E. Osteophytes form at joint margins F. Cartilage becomes softer and less elastic

3- A. Erosion of articular surfaces 5- B. Incongruity in joint surfaces 6- C. Reduction in motion 1- D. Joint cartilage becomes yellow and granular 4- E. Osteophytes form at joint margins 2- F. Cartilage becomes softer and less elastic

While the nurse is feeding a patient, the patient appears to choke on the food. Which symptoms indicate to the nurse that the patient has a partial airway obstruction? (select all that apply) A. Stridor B. Cyanosis C. Wheezing D. Bradycardia E. Rapid resp. rate

A, B, C Other indicators of partial airway obstruction: use of accessory muscles suprasternal and intercostal retraction flaring nostrils restlessness tachycardia change in LOC

In planning the care for a patient with a tracheostomy who has been stable and is to be discharged later in the day, the RN may delegate which interventions to the LPN? (select all that apply) A. Suction the tracheostomy B. Provide trach care C. Determine need for suctioning D. Assess the patient's swallowing ability E. Teach the patient about home tracheostomy care.

A, B, C a speech therapist does the swallow ability and the RN is responcible for all eduation

To what is the increase in emerging and untreatable infections attributed (select all that apply)? A. The evolution of new infectious agents B. use of antibiotics to treat viral infections C. Human population encroachment into wilderness areas D. Transmission of infectious agents from humans to animals E. An increased number of immunosuppressed and chronically ill people

A, B, C, E - HIV & Hanta virus have evolved to affect humans through closer association with animals as the human population encroaches into wilderness area. - Transmission of infections agents from animals to humans happens via west mile or Zika

During assessment of the patient with protein-calorie malnutrition, what should the nurse expect to find (select all that apply)? A. Frequent cold symptoms B. Decreased bowel sounds C. Cool, rough, dry scaly skin D. A flat or concave abdomen E. Prominent bony structure F. Decreased reflexes and lack of attention

A, B, C, E, F In malnutrition, metabolic processes are slowed, leading to increased sensitivity to cold, decreased HR and CO, and decreased neurologic function. Because of slow GI motility and absorption, the abdomen becomes distended and protruding and bowel sounds decrease. Skin is rough, dry and scaly where as bone structures protrude due to muscle loss. Because the immunity is impaired, there is a higher liklihood of infections

Which nursing actions are completed by the scrub nurse (select all that apply)? A. Prepares instrument table B. Documents intraoperative care C. Remains in the sterile area of the OR D. Checks mechanical and electrical equipment E. passes instruments to surgeon and assistant F. Monitors blood and other fluid loss and urine output

A, C, E

The nurse evaluates that patient teaching about a high calorie, high-protein diet has been effective when the patient selects which breakfast option from the hospital menu? A. 2 poached eggs, hash browns and whole milk B. 2 slices of toast with butter and jelly, orange juice and skim milk C. 3 pancakes with butter and syrup, 2 slices of bacon and apple juice D. cream of wheat with 2 tbsp of skim milk powder, one half grape fruit and a high-protein milkshake

A. 2 poached eggs, hash browns and whole milk Correct because 2 eggs= 24 g of protein and whole milk meats the calorie requirements Bacon is considered a fat rather than a meat servin

A patient taking Ibuprofen (Motrin) for treatment of OA has good pain relief but is experiencing increased dyspepsia and nausea with the drug's use. The nurse consults the primary care giver about doing what? A. Adding misoprostol (Cytotec) to the patient's drug regimen B. Substituting Naproxen for the Ibuprofen C. Returning to the use of acetominophen but at a dose of 5 g/day instead of 5 g/day D. Administering the ibuprofen with antacids to decrease the GI irritation

A. Adding misoprostol (Cytotec) to the patient's drug regimen Side effects of NSAIDs: - GI irritation and bleeding - dizziness -rash - headaches - tinnitus Misoprostol- used to prevent NSAID-induced gastic ulcers and gestritis. Naproxen has same side effects as ibuprofen and the daily dose of ibuprofen should not exceed 4g/day due to risk of liver damage.

What is the primary advantage of the use of midazolam (Versed) as an adjunct to general anesthesia? A. Amnestic effect B. Analgesic effect C. Prolonged action D. Antiemetic effect

A. Amnestic effect

A patient's tracheostomy tube becomes dislodged with vigorous coughing. What should be the nurse's first action? A. Attempt to replace tube B. Notify the health care provider C. Place the patient is high Fowler's position D. Ventilate the patient with a manual resuscitation bag until the health care provider arrives.

A. Attempt to replace tube If a tracheostomy tube is dislodges, the nurse should immediately attempt to replace the tube by grasping the retention sutures and spreading the opening. The obturator is inserted in the replacement tube, water soluble lubricant is applied to the tip, and the tube is inserted into the stoma at a 45 degree angle. The obturator is immediately removed to provide an airway. If the tube cannot be reinserted, the health care provider should be called and the patient should be assessed for the level of respiratory distress, positioned in semi-fowlers and ventilated with a manual resuscitation bag only if necessary until assistance arrives.

What is the primary goal of the circulating nurse during preparation of the operating room, transferring and positioning the patient, and assisting the anesthesia team? A. Avoiding any type of injury to the patient B. Maintaining a clean environment for the patient. C. Providing for patient comfort and sense of well-being D. Preventing breaks in aseptic techniques by the sterile members of the team

A. Avoiding any type of injury to the patient

What problem should the nurse assess the patient for if the patient was on prolonged antibiotic therapy? A. Coagulation problems B. Elevated serum ammonia levels C. Impaired absorption of amino acids D. Increased mucus and bicarbonate secretions

A. Coagulation problems Bacteria in the colon synthesize Vitamin K, which is needed for the production of prothrombin by the liver.

What does progression of patients through various phases of care in a PACU primarily depend on? A. Condition of patient B. Type of anesthesia used C. Preference of surgeon D. Type of surgical procedure

A. Condition of patient

What is the pathophysiology of systemic lupus erythematous (SLE) characterized by? A. Destruction of nucleid acids and other self-proteins by antibodies B. Overproduction of collagen that disrupts the functioning of internal organs C. Formation of abnormal IgG that attaches to cellular antigens, activating complement D. Increased activity of T suppressor cells with B-cell hypoactivity resulting in an immunodeciciency

A. Destruction of nucleid acids and other self-proteins by antibodies This is a hypersensitive response, NOT an immunodeficiency

The nurse is caring for a patient receiving 1000 mL of parenteral nutrition solution over 24 hours. When it is time to change the solution, 150 mL remain in the bottle. What is the most appropriate action by the nurse? A. Hang the new solution and discard the unused solution B. Open the IV line and rapidly infuse the remaining solution C. Notify the health care provider for instructions regarding the infusion rate D. Wait to change the solution until the remaining solution infuses at the prescribed rate

A. Hang the new solution and discard the unused solution Bacterial growth occurs at room temperature. Therefore solutions MUST not infuse for more than 24 hours Speeding up the solution may cause hyperglycemia

A 32-year-old man is admitted to the hospital with an acute exacerbation of Crohn's disease. Coping strategies that might be suggested by the nurse during his hospitalization include (select all that apply) A. Humor B. Exercise C. Journaling D. A cleansing diet E. Relaxation therapy

A. Humor C. Journaling E. Relaxation therapy

A paient in the PACU has emergence delirium manifested by agitation and thrashing. What should the nurse assess for first in the patient? A. Hypoxemia B. Neurological Injury C. Distended Bladder D. Cardiac Dysrhythmias

A. Hypoxemia

With what are the postoperative respiratory complications of atelectasis and aspiration of gastric contents associated? A. Hypoxemia B. Hypercapnia C. Hypoventilation D. Airway Obstruction

A. Hypoxemia

Number the stages of the general adaptation syndrome (GAS) according to the signs and symptoms the nurse would expect to see. Use 1 for the Alarm Stage; 2 for the Resistance stage and 3 for the Exhaustion stage A. Increased Agitation B. Increased HR C. Continuous viral infections D. Bleeding ulcer E. Increased blood glucose levels F. No sign or Symptoms evident

A. Increased Agitation - 1 B. Increased HR - 1 C. Continuous viral infections - 3 D. Bleeding ulcer - 3 E. Increased blood glucose levels - 1 F. No sign or Symptoms evident - 2

When planning nutritional interventions for a healthy 83-year-old man, the nurse recognizes what factor is most likely to affect his nutritional status. A. Living alone on a fixed income B. Changes to cardio function C. An increase in GI motility and absorption D. Snacking between means, resulting in obesity

A. Living alone on a fixed income

Which description is most characteristic of OA when compared to RA? A. Not systemic or symmetric B. Rheumatoid factor positive C. Most commonly occurs in women D. Morning joint stiffness lasts one to several hours

A. Not systemic or symmetric OA- not systemic or symmetric. Morning joint stiffness resolves in 30 min RA- is RF positive and characterized by being systemic and affecting small joints symmetrically. Morning joint stiffness lasts 1 hour to all day

The PACU nurse applies warm blankets to a postoperative patient who is shivering and has a body temperature of 90.0 F. What treatment also maybe used to treat the patient? A. Oxygen B. Vasodilating drugs C. Antidysrhythmic drugs D. Analgesics or sedatives

A. Oxygen

A health care provider diagnosis a patient with a plantar wart. What should the nurse know about this kind of abnormality? A. Papilloma growth of the sole of the foot B. Thickening of skin on the weight bearing part of foot C. Local thickening of skin caused by pressure on bony prominences D. Tumor on nerve tissue between 3rd and 4th metatarsals.

A. Papilloma growth of the sole of the foot Callus- Thickening of skin on the weight bearing part of foot Corn- Local thickening of skin caused by pressure on bony prominences Morton's neuroma- tumor on nerve tissue between 3rd and 4th metatarsals.

What should the nurse teach the patient recovering from an episode of acute back pain? A. Perform daily exercise as a lifelong routine B. Sit in a chair with hips higher than knees C. Avoid occupations in which the use of the body is required D. Sleep on the abdomen or on the back with legs extended.

A. Perform daily exercise as a lifelong routine

A patient develops epistaxis upon removal of a NG tube. What action should the nurse take? A. Pinch the soft part of the nose B. Position the patient on the side C, have the patient hyperextend the neck D. Apply an ice pack to the back of the neck

A. Pinch the soft part of the nose Epitaxis- nose bleed. Put head forward and keep pressure on nose for 10-15 minutes

The patient has been diagnosed with an early vocal cord malignancy. The nurse explains that usual treatment includes: A. Radiation therapy that preserves the quality of voice B. a hemilaryngectomy that prevents the need for a tracheostomy C. A radical neck dissection that removes possible sites of metastasis D. a total laryngectomy to prevent development of second primary cancers.

A. Radiation therapy that preserves the quality of voice If laryngeal tumors are small, radiation is the treatment of choice. Surgical treatments are used only if larger lesions are present or radiation doesn't work

While teaching relaxation therapy to a patient with fibromyalgia, what does the nurse recognize as being most important to incorporate? A. Relaxation breathing B. Soft background music C. Progressive muscle relaxation D. Concentration on a single focus

A. Relaxation breathing

Identify 4 personal characteristics that promote adaptation to stressors.

A. Resilience B. Hardiness C. Attitude D. Optimism

.During the healing phase of inflammation, which cells would be most likely to regenerate? A. Skin B. Neurons C. Cardiac Muscle D. Skeletal Muscle

A. Skin

Identify the behaviors listed below as either positive coping (P) or negative coping (N) strategies. A. Smoking cigarettes B. Ignoring a situation C. Joining a support group D. Starting an exercise program E. Increasing time spent with friends

A. Smoking cigarettes (N) B. Ignoring a situation (N) C. Joining a support group (P) D. Starting an exercise program (P) E. Increasing time spent with friends (P)

A patient with OA asks the nurse whether he could try some glucosamine and chondoitin for control of his symptoms. The best response by the nurse includes what information? A. Some patients find these supplements helpful for relieving arthritis pain and improving mobility B. Although these substances may not help, there is no evidence that they can cause negative effects C. These supplements are a fad that has not been shown to reduce pain or increase joint mobility in patient's with OA D. Only dosages of these supplements available by prescription are high enough to provide any benefit in treatment of OA

A. Some patients find these supplements helpful for relieving arthritis pain and improving mobility These drugs should be discontinues if no improvement is seen over 90-120 days of consistent use These drugs may decrease the effectiveness of antidiabetic drugs and increase the risk of bleeding

What contributes to increased protein-calorie needs? A. Surgery B. Vegan Diet C. Lowered temperaturee D. Cultural or religious beliefs

A. Surgery A patient will recover more quickly with a balanced nutritional status before the surgery and increased protein in needed for the healing process after surgery. None of the other options would alter the need for protein.

After receiving the assigned patients or the day, the nurse determines that stress-relieving interventions are a priority for which patient? A. The man with peptic ulcer disease B. The newly admitted women with cholecystitis C. The man with a bacterial exacerbation of chronic bronchitis D. The woman who is 1 day post op for a knee replacement

A. The man with peptic ulcer disease

What characterizes auscultation of the abdomen? A. The presence of borborygmi indicates hyperperistalsis B. The bell of the stethoscope is used to auscultate high-pitched sounds C. Hihg-pitched, rushing and tinkling bowel sounds are heard after eating D. Absence of bowel sounds for 1 minute in each quadrant is reported as abnormal

A. The presence of borborygmi indicates hyperperistalsis Borborygmi are loud gurgles. Normal stomach sounds are high pitched and are heard best with the diaphragm of the stethoscope. High pitched tinkling sounds occur with bowel obstruction. No bowel sounds for 2-3 minutes are reported as abnormal

When considering tube dealings for a patient with severe protein-calorie malnutrition, what is an advantage of a gastronomy tube vs. a nasogastric tube? A. There is less irritation to the nasal and esophageal mucosa B. The patient experiences the sights and smells associated with eating C. Aspiration resulting from reflux of formulas into the esophagus is less common D. Routine checking for placement is not required because gastronomy tubes do not become displaced

A. There is less irritation to the nasal and esophageal mucosa NG tube should only be used for short term feeding problems because prolonged therapy can result in irritation and erosion of mucosa of the upper GI tract. Gastric reflux can occur with both and both can become dislodged.

A patient has recently had a myocardial infarction. What emotion-focused coping strategies should the nurse encourage him to use to adapt to the physical and emotional stress of his illness (Select all that apply)? A. Use Medication B. Plan dietary changes C. Starting an exercise program D. Do favorite escape activities (e.g. playing cards) E. Share feelings with souse or other family memebers

A. Use Medication (should this be meditation) D. Do favorite escape activities (e.g. playing cards) E. Share feelings with souse or other family memebers

When assessing that patient on return to the surgical unit following a total larygectomy and radical neck dissection, what would the nurse expect to find? A. a closed wound drainage system B. a nasal endotracheal tube in place C. a NG tube with orders for tube feedings D. a tracheostomy tube and mechanical ventilation

A. a closed wound drainage system - the patient does have NG tube, but is used for suctioning and will later be used for feedings - they may have a tracheostomy tube in place, but they will not be on mechanical ventilation

A break in sterile technique occurs during surgery when the scrub nurse touches... A. the mask with sterile gloved hands B. sterile gloved hands to the gown at chest level C. the drape at the incision site with sterile gloved hands D. The lower arm to the instrument tray

A. the mask with sterile gloved hands

A patient is started on aldendronate (Fosamax) once weekly for the treatment of osteoporosis. The nurse determines that further instruction about the drug is needed when what is said by the patient? A." I should take the drug with a meal to prevent stomach irritation" B. "This drug will prevent further bone loss and increase my bone density" C. "I need to sit or stand upright for at least 30 minutes after taking this drug" D. "I will still need to take my calcium supplements while taking this drug"

A." I should take the drug with a meal to prevent stomach irritation" the bisphosphonates such as alendronate, must be taken at least 30 minutes before food or other medications to promote absorption. Because they are irritating to the stomach, the patient should remain upright for 30 min to limit reflux. Calcium and Vit D is still needed for bone formation.

Consider the differences between primary and secondary prevention. Fill in the blanks: Actions aimed at early detection of disease and interventions to prevent progression of the disease are considered _______A_____ prevention. Following a proper diet, getting appropriate exercise, and receiving immunizations against specific diseases is considered _____B_____ prevention

A= Secondary B= Primary

List three behaviors that can contribute to insomnia

Any of the following can contribute: Consumption of stimulants close to bedtime Side effect of medications Drinking alcohol or using OTC medications as a sleep aid Long afternoon naps Sleeping in late Exercise near bedtime Jet Lag Nightmares Stressful Life Event Medical conditions or psychiatric illnesses Irregular sleep/wake schedule worry about getting enough sleep

The old-old population (over 85 yrs old) has an increased risk for frailty. However, old age is just one element of frailty. Identify at least three other assessment findings that contribute to frailty.

Any of the following: Unplanned weight loss (more than 10 pounds in a year) Weakness Poor Endurance and Energy Slowness Low Activity

The nurse in a clinic is talking with a patient who will be traveling from the Midwest Time Zone to Moscow to attend a 4-day conference. The patient asks the nurse how he can minimize the effects of jet lag. What are at least two recommendations that the nurse could give the patient?

Any of these responses would be appropriate: Start to get in harmony with Moscow time zone several days before you travel Be sure to expose yourself to daytime daylight, which will assist with synchronizing your body's clock to environmental time Melatonin has been shown to be an effective sleep aid to help synchronize the body's rhythym Resynchronization of the body's clock will occur at a rate of 1 hr/day if the patient travels eastward.

Nurses who rotate shifts or work nights are at risk for developing shift work sleep disorder characterized by insomnia, sleepiness, and fatigue. Identify at least three negative implications for the nurse.

Any three of these would be correct: Nurse is too sleepy to be fully awake at work Nurse is too alert to sleep soundly the next day Increased morbidity and mortality related to cardiovascular problems Mood disorders are higher Fatigue could result in errors and accidents, as perceptual skills, judgment, and decision-making abilities may be diminished.

Identify methods to specifically prevent osteoporosis in postmenopausal women? ( select all that apply) A. Eating beef B. Eating 8 ounces of yogurt a day C. Performing weight-bearing exercise D. Spending 15 minutes in the sun each day E. Taking postmenopausal estrogen replacement

B, C To prevent osteoporosis: Increased calcium or Vit D intake weight bearing exercises

Which female patients are at risk for developing osteoporosis? A. 60 year old white aerobics teacher B. 55 year old Asian American cigarette smoker C. 62 year old African American on estrogen therapy D. 68 year old while who is underweight and inactive E. 58 year old Native American who started menopause prematurly

B, D, E Risk Fators: age over 65 white or Asian ethnicity cigarette smoking inactive lifestyle low body weight premature menopause

For which nursing diagnosis or collaborative problems common in postoperative patients has ambulation been found to be appropriate intervention (select all that apply)? A. Impaired skin integrity related to decreased muscle strength B. Impaired mobility related to decreased muscle strength C. risk for aspiration related to decreased muscle strength D. Ineffective airway clearance related to decreased respiratory excursion E. Constipation related to physical activity and impaired GI motility F. Venous thomboembolism related to dehydration, immobility, vascular manipulation, or injury

B, D, E & F

After teaching a patient with RA about the prescribed theraputic regimen, the nurse determines that further instruction is needed when the patient says what? A. " It is important for me to perform my prescribed exercises every day" B. "I should perform most of my daily chores in the morning when my energy level is highest" C. "An ice pack to a joint for 10 min. may help to relive pain and inflammation when I have an acute flare." D. " I can use assistive devices such as padded utensils, electric can openers and elevated toilet seat to protect my joints.

B. "I should perform most of my daily chores in the morning when my energy level is highest"

What is the most normal functioning method of speech restoration in the patient with a total laryngectomy? A. Esophageal speech B. A transesophageal puncture C. An electrolarynx D. An electolarynx place in the mouth

B. A transesophageal puncture Transesophageal puncture- most normal voice, but requires a surgical fistula made between the esophagus and the trachea, with, maybe, a valve prosthesis Esophageal speech- involves trapping air in the esophagus and releasing it to form sound but only 10% of patients can develop fluent speech Electrolarynx- whether in mouth or neck, allows speech that is robotic

What type of dressing will the nurse most likely use for the patient with a stage II pressure ulcer. this wound is classified as a yellow wound using the red-yellow-black concept of wound care. A. Dry, sterile dressing B. Absorptive dressing C. Negative pressure wound therapy D. Telfa dressing with antibiotic ointment

B. Absorptive dressing

A patient had abdominal surgery 3 months ago and calls the clinic with complaints of severe abdominal pain and cramping, vomiting and bloating. What should the nurse most likely suspect as the cause of the patient's problem? A. Infection B. Adhesion C. Contracture D. Evisceration

B. Adhesion

Because of the rapid elimination of volatile liquids used for general anesthesia, what should the nurse anticipate the patient will need early in the anesthesia recovery period? A. Warm blankets B. Analgesic Medication C. Observation for respiratory depression D. Airway protection in anticipation of vomiting

B. Analgesic Medication

A 92-year old female patient is being admitted to the emergency department with severe shortness of breath. Being aware of the patient's condition, what approach should the nurse use to assess the patient's lungs? (select all that apply) a. Apex to Base b. Base to apex c. Lateral sequence d. anterior then posterior e. posterior then anterior

B. Base to Apex E. Posterior then Anterior

The patient has had RA for some time bt has not had success with previous medication. Although there is an increased risk for tuberculosis, which monoclonal antibody is used with methotrexate to best treat symptoms? A. parenteral gold B. Certolizumab (Cimzia) C. Tocilizumab (Actemra) D. Hydroxychloroquine (Paquenil)

B. Certolizumab (Cimzia)

During treatment of a patient with an acute gout attack, the nurse would expect to administer which drug? A. Asprin B. Cholchicine C. Allopurinol (Zyloprim) D. Probenecid (Benemid)

B. Cholchicine

While caring for a female patient with Alzheimer's disease and her caregiver husband, the nurse finds that the patient's husband is experiencing increased asthma problems. What is a possible explanation for this finding? A. Progressive worsening of asthma occurs in people as they age. B. Chronic and intense stress can cause exacerbation of immune-based diseases C. The husband is probably smoking more to help him cope with needing to care continually for his wife D. The husband inadequately copes with his wife's condition by unconsciously forgetting to take his medication

B. Chronic and intense stress can cause exacerbation of immune-based diseases

What is included in the routine assessment of the patient's cardiovascular function on admission to the PACU? A. Monitoring arterial blood gases B. ECG monitoring C. Determining fluid and electrolyte status D. Direct arterial blood pressure monitoring

B. ECG monitoring

When teaching the older adult about nutritional needs during aging, what does the nurse emphasize? A. Need for all nutrients decrease as you age B. Fewer calories, but the same or slightly increased amount of protein, are requires as one ages C. Fats, Carbs and Protein should be decreased, but vitamin and mineral intake should increase D. High-calorie oral supplements should be taken between meals to ensure that recommended nutrient needs are met.

B. Fewer calories, but the same or slightly increased amount of protein, are requires as one ages Calorie intake should be decreased due to lesser activity and basal metabolic rate the need for specific nutrients (such as proteins and vitamins do not change)

A patient with chronic osetomyelitis has been hospitalized for a surgical debridement procedure. What does the nurse explain to the patient as the rationale for the surgical treatment? A. Removal of the infection prevents the needs for bone and skin grafting B. Formation of scar tissue has led to a protected area of bacterial growth C. The process of depositing new bone blocks the vascular supply to the bone D. Antibiotics are not effective against microorganisms that cause chronic osteomyelitis

B. Formation of scar tissue has led to a protected area of bacterial growth Chronic infection in the bone leads to the formation of scar tissue from the granulation tissue. The avascular scar provides an ideal site for microorganisms and is impenetrable to antibiotics. Surgical debriedment removes the scar tissue and dead bone and allows them to instill antibiotics directly into the affected area. However, bone and skin grafting may be necessary later depending on the extent of destruction

A patient with osteomyelitis has a nursing diagnosis of risk for injury. What is an appropriate nursing intervention for this patient? A. Use careful and appropriate disposal of soiled dressings. B. Gently handle the involved extremity during movement. C. Measure the circumference of the affected extremity daily. D. Provide ROM exercise q4h to the involved extremity.

B. Gently handle the involved extremity during movement. a patient with osteomyelitis has risk for pathogenic fractures at the site of the infection. Appropriate disposal fo dressings is important, but not directly related to this patient. Edema is not related to this disorder ROM with be decreased due to risk of infection and fracture.

A patient with gout is treated with drug therapy to prevent future attacks. The nurse teaches the patient that what is most important for the patient to do? A. Avoid all foods high in purine, such as organ meats B. Have periodic determination of serum uric acid levels C. Increase dosage of medication with the onset of an acute attack. D. Perform active ROM of all joints that have been affected by gout

B. Have periodic determination of serum uric acid levels - with use of medication, dietary restriction of acohol and meats are usually not necessary - joint immobilization is used for acute gout attacks

When transporting an inpatient to the surgical department, which area is a nurse from another area of the hospital able to access? A. Clean core B. Holding area C. Corridors of surgical suite D. unprepared operating room

B. Holding area

What should the nurse include in discharge teaching for the patient with a total laryngectomy? A. How to use esophageal speech to communicate B. How to use a mirror to suction the tracheostomy C. The necessity of never covering the laryngectomy stoma D. The need to use baths instead of showers

B. How to use a mirror to suction the tracheostomy - Teaching voice rehabilitation is done by a speech therapist, but the nurse can talk to them about different options - A laryngectomy stoma should be covered with a shield during showering and covered when there is a risk of aspirating foreign materials

What condition should the nurse anticipate that night occur during epidural and spinal anesthesia. A. Spinal headache B. Hypotension and bradycardia C. Loss of consciousness and seizures D. Downward extension of nerve block

B. Hypotension and bradycardia

An 82-year-old male patient with pneumonia who is in the ICU in beginning to have decreased cognitive function. What should the nurse first suspect as a potential cause of this change? A. Fatigue B. Infection C. ICU psychosis D. Medication allergy

B. Infection

What is the physical environment of a surgery suit designed to promote? A. Electrical safety B. Medical and surgical asepsis C. Comfort and privacy of patient D. Communication among the surgical team

B. Medical and surgical asepsis

What is included in the nursing care of the patient with a cuffed tracheostomy tube? A. Change tube every 3 days B. Monitor cuff pressure every 8 hours C. Perform mouth care every 12 hours D. Assess arterial blood gases every 8 hours

B. Monitor cuff pressure every 8 hours

What is a normal finding on physical examination of the abdomen? A. Auscultation of buits B. Observation of visible pulsations C. Percussion of liver dullness in the left midclavicular line D. Palpation of the spleen 1-2 cm below the left costal margin

B. Observation of visible pulsations buits are abnormal. liver is percussed at the RIGHT midclavicular line and a normal spleen cannot be palpated

To preserve function and the ability to perform ADLs, what should the nurse teach the patient with OA? A. Avoid exercise that involves the affected joints B. Plan and organize task performance to be less stressful to joints C. Maintain normal activities during an acute episode to prevent loss of function D. Use mild analgesics to control symptoms when performing tasks that cuase pain

B. Plan and organize task performance to be less stressful to joints Principles of joint protection: - patients should be helped to perform tasks with less stress - ROM, isotonic and isometric exercises of the affected joints should be balanced with rest and protection - During an acute episode, joints should be rested - Joints should be used only to the point of pain and masking pain with analgesics can cause damage

Which short-acting barbiturates are most commonly used for induction of general anesthesia (select all that apply)? A. Nitrous oxide B. Propofol (Diprivan) C. Isoflurane (Florane) D. Thioprental sodium (Pentothal) E. Sodium methohexital (Drevital)

B. Propofol (Diprivan) E. Sodium methohexital (Drevital)

What role do the B-complex vitamins play in wound healing? A. Decreased metabolism B. Protect protein from being used for energy C. Provide metabolic energy for the inflammatory process D. Coenzymes for fat, protein and carbohydrate metabolism

B. Protect protein from being used for energy

A 24-year-old patient with a 12-year history of muscular dystrophy is hospitalized with heart failure. What is an appropriate nursing intervention for this patient? A. Feed and bathe the patient to avoid exhausting the muscle B. Resposition frequently to avoid skin and respiratory complications C. Provide hand weights for the patient to exercise the upper extremities D. Use orthopedic braces to promote ambulation and prevent muscle wasting

B. Resposition frequently to avoid skin and respiratory complications Promotion of muscle activity is important in any patient with muscular dystrophy, but when the disease has progressed to cardiomyopathy or respiratory failure, activity must be balanced with oxygen supply. At this stage of the disease, care should be taken to prevent skin or respiratory complications. The patient should be encouraged to perform as much self-care and exercise as energy allows but this will be limited.

Before administering a bolus of intermittent tube feelings to a patient with a percutaneous endoscopic gastronomy (PEG), the nurse aspirates 220 mL of gastric contents. How should the nurse respond? A. Return the aspirate t the stomach and recheck the volume of aspirate in an hour B. Return the aspirate to the stomach and continue with the tube feeding as planned. C. Discard the aspirate to prevent overspending the stomach when the new feeding is given. D. Notify the health care provider that the feedings have been schedules too frequently to allow for stomach emptying

B. Return the aspirate to the stomach and continue with the tube feeding as planned. With intermittent feedings, less than 250 mL residual does not require further action. A promotility agent should be considered with continuous feedings and a residual of 250 mL or more after the second residual check.

A preoperative patient reveals that an uncle died during surgery because of a fever and cardiac arrest. Knowing the patient is a risk for malignant hyperthermia, the perioperative nurse alerts the surgical team. What is likely to happen next? A. The surgery will have to be cancelled B. Specific precautions can be taken to safely anesthetize the patient. C. Dantrolene (Dantrium) must be given to prevent hyperthermia during surgery. D. The patient should be placed on a cooling blanket during the surgical procedure.

B. Specific precautions can be taken to safely anesthetize the patient.

What is characteristic of chronic inflammation? A. It may last 2 - 3 weeks B. The injurious agent persists or repeatedly injured tissue C. Infective endocarditis is an example of chronic inflammation D. Neutrophils are the predominant cell type at the site of inflammation.

B. The injurious agent persists or repeatedly injured tissue

When a patient at the clinic is informed that testing indicates the presence of gonorrhea, the patient sighs and says "That, i can handle." What does the nurse understand about the patient in this situation? A. The patient is in denial about the possible complications of gonorrhea B. The patient does not perceive the gonorrhea infection as a serious stressor. C. The patient does not have other current stressors that require adaptation or coping mechanisms D. The patient knows how to cope with gonorrhea from dealing with previous gonorrhea infections

B. The patient does not perceive the gonorrhea infection as a serious stressor.

Which statement best describes osteosarcoma? A. High rate of local recurrence B. Very malignant and metasticizes early C. Arises in cancellous ends of long bones D. Develops in the medullary cavity of long bones

B. Very malignant and metastasizes early It is the most common primary bone cancers. Often brought to attention by injury A. High rate of local recurrence - Osteoclastoma C. Arises in cancellous ends of long bones- Osteoclastoma D. Develops in the medullary cavity of long bones- Ewing's Sarcoma

To prevent agitation during the patient's recovery from anesthesia, when should the nurse begin orientation explanations? A. When the patient is awake B. When the patient first arrives in the PACU C. When the patient becomes agitated or frightened D. When the patient can be aroused and recognizes where he or she is

B. When the patient first arrives in the PACU

During a follow-up visit to a patient with acute osteomyelitis treated with IV antibiotics, the home health nurse is told by the patient's wife that she can hardly get the patient to eat because his mouth is so sore. In assessing the patient's mouth, what should the nurse expect to find? A. A dry, cracked tongue with a central furrow B. While, curdlike membranous lesions of the mucosa C. Ulcers of the mouth and lips surrounded by a reddened base D. Single or clustered vesicles on the tonfge and buccal mucosa

B. While, curdlike membranous lesions of the mucosa One of the most common adverse effects of prolonged ans high dose antibiotics is an overgrowth of Candida albicans in the oral and genital cavity. A dry, cracked tongue with a central furrow indicates dehydration. Single or clustered vesicles on the tongue and buccal mucosa indicates HSV Ulcers of the mouth and lips surrounded by a reddened base indicate canker sores or aphthous somatitis

A patient is admitted to the hospital with left upper quadrant pain. What may be a possible source of the pain? A. Liver B. pancreas C. Appendix D. Gallbladder

B. pancreas Pancreas = LUQ Liver = RUQ Appendix = RLQ Gallbladder = RUQ

A laminectomy and spinal fusion are performed on a patient with a herniated lumbar intervertebral disc. During the postoperative period, which finding is of most concern to the nurse? A. Paralytic ileus B. urinary incontinence C. Greater pain at the graft site than at the lumber incision site. D. Leg and arm movement and sensation unchanged from the preoperative status

B. urinary incontinence urinary incotinence could be a sign of nerve damage and should be reported to the health care provider. All other answers normal following surgery

Which digestive substances are activated in the stomach (select all that apply)? A. Bile B.. Pepsin C. Gastrin D. Maltese E. Secretin F. Amylase

B.. Pepsin C. Gastrin Bile = gallbladder Maltese = secreted in small intestine and converts maltose to glucose Secretin= secreted by duodenal mucosa Amylase= secreted by small intestine and pancreas

Which tubes drain gastric contents? (select all that apply) A. T-tube B. Hemovac C. NG tube D. Indwelling catheter E. GI tube

C, E

What are the characteristics of Paget's Disease? (select all that apply) A. Results from Vit D deficiency B. Loss of total bone mass and substance C. Abnormal remodeling and reabsorption of bone D. Most common i bones of spine, hips and wrists E. Generalized bone decalcification with bone deformity F. Replacement of normal marrow with vascular connective tissue

C, F Osteoperosis- Loss of total bone mass and substance Osteoperosis- Abnormal remodeling and reabsorption of bone Osteoperosis- D. Most common i bones of spine, hips and wrists Osteomalacia- Results from Vit D deficiency Osteomalacia- Generalized bone decalcification with bone deformity

Following 2 weeks of IV antibiotic therapy, a patient with acute osteomyelitis of the tinia is preparing for discharge from the hospital. The nurse determines that additional instruction is needed when the patient makes which statement? A. "I will need to continue my antibiotic therapy for 4-6 weeks" B. "I shouldn't bear weight on my affected leg until healing is complete." C. "I can use a heating pad on my lower leg for comfort and to promote healing." D. "I should notify the health care provider if the pain in my leg becomes worse."

C. "I can use a heating pad on my lower leg for comfort and to promote healing." Activities such as exercise or heat application, which increases circulation serve to stimulate the spread of infection, should be avoided by patients with acute osteomyelitis. All other statements are true.

Which patient is at the greatest risk for developing pressure ulcers? A. A 42-year-old obese woman with type 2 diabetes B. A 78-year-old man who is confused and malnourished C. A 30-year-old man who is comatose following a head injury D. A 65-year-old woman who has urge and stress incontinence

C. A 30-year-old man who is comatose following a head injury

While assessing a patient in the PACU, the nurse finds that the patient's blood pressure is below preoperative baseline. The nurse determines that the patient has residual vasodilating effects of anesthesia when what is assessed? A. A urinary output greater than 30 mL/hr B. An oxygen saturation of 88% C. A normal pulse with warm, dry, pink skin D. A narrow pulse pressure with normal pulse

C. A normal pulse with warm, dry, pink skin

What is a normal finding during physical assessment of the mouth? A. A red, slick appearance of the tongue B. Vulvular deviation to the side on saying "Ahh" C. A thin white coating of the dorsum of the tongue D. Scattered red, smooth areas on the dorsum of the tongue

C. A thin white coating of the dorsum of the tongue

What is the advantage of a tracheostomy tube over an endotracheal tube for long-term management of upper airway obstruction? A. A tracheostomy is safer to perform in an emergency B. A ET tube has a higher risk of tracheal pressure necrosis C. A tracheostomy tube allows for more comfort and mobility D. An ET tube is more likely to read to lower resp. infection

C. A tracheostomy tube allows for more comfort and mobility a trach tube is more comfortable because there isn't a tube in the patient's mouth like with an ET tube. with a trach tube, there is more movement allowed because it is more secure. An ET tube is easier to insert in an ER.

How is the initial information given to the PACU nurses about the surgical patient? A. A copy of the written operative report B. A vertebral report from the circulating nurse C. A verbal report from the anesthesia care provider D. An explanation of the surgical procedure from the surgeon

C. A verbal report from the anesthesia care provider

What does the mechanism of chemotaxis accomplish? A. Causes the transformation of monocytes into macrophages B. Involves a pathway of chemical processes resulting in cellular lysis C. Attracts the accumulation of neutrophils and monocytes to an area of injury D. Slows the blood flow in a damaged area, allowing migration of leukocytes into tissue

C. Attracts the accumulation of neutrophils and monocytes to an area of injury

For which patient is ready for discharge from Phase I PACU care to the clinical unit? A. Arouses easily, pulse is 112 bpm, respiratory rate is 24, dressing is saturated, SaO2 is 88% B. Difficult to arouse, pulse is 52, RR is 22, dressing is dry and intact, SaO2 is 91% C. Awake, VS stable, dressing is dry and intact, no respiratory depression, SaO2 is 92% D. Arouses, BP higher than preoperative and RR is 10, no excessive bleeding, SaO2 is 90%

C. Awake, VS stable, dressing is dry and intact, no respiratory depression, SaO2 is 92%

A patient who experienced an open fracture of the humerus 2 weeks ago is having increased pain at the fraction site. To identify a possible causative agent of osteromyelitis at the site, what should the nurse expect testing to include? A. X-ray B. CT scan C. Bone biopsy D. WBC count and erythrocyte sedimentation rate (ESR

C. Bone biopsy because large doses of appropriate antibiotics are necessary in the treatment of acute osteomyelitis, it is important to identify the causative microorganism. The definitive way to find this is a bone biopsy. or a biopsy of the soft tissue surrounding the site. The other tests help establish the diagnosis but do not identify the causative agent.

Following a supraglottic laryngectomy, the patient is taught how to use the supraglottic swallow to minimize the risk of aspiration. In teaching the patient about this technique, what should the nurse instruct the patient to do? A. Perform Valsalva maneuver immediately after swallowing B. Breath between each Valsalva maneuver and cough sequence C. Cough after each swallow to remove food from the top of the vocal cords D. Practice swallowing thin, watery fluids before attempting to swallow solid foods.

C. Cough after each swallow to remove food from the top of the vocal cords Supraglottic laryngectomy- removal of the epiglottis and false vocal cords. The removal of the epiglottis allows food to enter trachea. Supraglottic Swallow- take deep breath, put food or fluid in mouth, swallow while holding breath, cough immediatley after swallowing. Swallow again, then breath. The valsalva maneuver should be performed BEFORE putting food in mouth

A patient relcieving atropine, an anticholinergic drug, in preparation for surgery. the nurse expects this drug to affect the GI tract by doing what? A. Increasing gastric emptying B. Relaxing pyloric and ileocecal sphincters C. Decreasing secretions and peristaltic action D. Stimulating the nervous system of the GI tract

C. Decreasing secretions and peristaltic action The parasympathetic nervous system stimulates activity in the GI system (increasing motility and secretions and relaxing sphincters to promote movement of contents). Anticholinergic drug drug blocks this activity

A patient's documentation indicates he has a stage II pressure ulcer on his right hip. What should the nurse expect to find on assessment of the patient's right hip? A. Exposed bone, tendon or muscle B. An abrasion, blister or shallow crater C. Deep crater through subcutaneous tissue to fascia D. Persistent redness (or bluish color in darker skin tones)

C. Deep crater through subcutaneous tissue to fascia

A 70-year-old patient is being evaluated for symptoms of RA. The nurse recognizes what as the major problem in the management of RA in the older adult? A. RA is usually more severe in older adults B. Older patients are not as likely to comply with the treatment regimens C. Drug interactions and toxicity are more likely to occur with multidrug therapy D. Laboratory and other diagnostic tests are not effective in identifying RA in older adults.

C. Drug interactions and toxicity are more likely to occur with multidrug therapy

What is an ominous sign of advanced SLE? A. Proteinuria from early glomerulonephritis B. Anemia from antibodies against blood cells C. Dysrhythmias from fibrosis of the atrioventricular node D. Cognitive dysfuction from immune complex deposit in the brain

C. Dysrhythmias from fibrosis of the atrioventricular node

A patient who had a major surgery is experiencing emotional stress as well as physiologic stress from the effects of surgery. What can this stress cause? A. Diuresis B. Hyperkalemia C. Fluid retention D. Impaired blood coagulation

C. Fluid retention

What may occur with failure of the sodium-potassium pump during severe protein depletion? A. Ascites B. Anemia C. Hyperkalemia D. Hypoalbuminium

C. Hyperkalemia The sodium potassium pump uses 20-50% of all calories ingested. When energy sources are decreased, the pump fails to function, sodium and water are left in the cell and potassium remains in extracellular fluid. Hyperkalemia, as well as hyponatremia, can occur.

In promoting healthy feet, what factor should the nurse recognize is associated with most foot problems? A. Poor foot hygiene B. Congenital Deformities C. Improperly fitting shoes D. Peripheral vascular disease

C. Improperly fitting shoes

The nurse monitors the lab results of the patient with protein-calorie malnutrition during treatment. Which results in an indication of improvement in the patient's condition. A. Decreased lymphocytes B. Increased serum ptassium C. Increased serum transferrin D. Decreased serum prealbumin

C. Increased serum transferrin Serum transferrin is a protein that is synthesized by the liver and used for iron transport and decreases when there is protein deficiency. An increase in the protein would indicate a more positive nitrogen balance with amino acids available for synthesis. Decreased lymphocytes and serum prealbumin are indicators of protein depletion and increased serum potassium shows continuing failure of the sodium-potassium pump

When caring for a patient who has had most of the stomach surgically removed, what is important for the nurse to teach the patient? A. Extra iron will need to be taken to prevent anemia B. Avoid foods with lactose to prevent bloating and diarrhea C. Lifelong supplementation of cobalamin (Vitamin B12) will be needed D. Because of the absence of digestive enzymes, protein malnutrition is likely.

C. Lifelong supplementation of cobalamin (Vitamin B12) will be needed The stomach secretes intrinsic factor, which is required for B12 absorption in the intestine.

What is an indication for perenteral nutrition that is not an appropriate indication for enteral tube feedings? A. Head and neck cancer B. Hypermetabolic state C. Malabsorption syndrome D. Protein-calorie malnutrition

C. Malabsorption syndrome Enteral- using GI tract to absorb food parenteral- delivery of nutrients and calories into a vein

During surgery, a patient has nursing diagnosis of risk for perioperative positioning injury. What is a common risk factor for this nursing diagnosis. A. Skin lesions B. Break in sterile techniques C. Musculoskeletal deformities D. Electrical or mechanical equipment failure

C. Musculoskeletal deformities

Before repositioning the patient on their side after a lumbar laminectomy, what should the nurse's first action be? A. Raise head of bed to 30 degrees B. Have the patient flex the knees and hips C. Place a pillow between patient's legs D. have the patient grasp the side rail on the opposite side of the bed.

C. Place a pillow between patient's legs After spinal surgery, you want to keep the patient's spin in alignment as much as possible. Putting a pillow between their legs and moving the body as a unit is best.

What is the primary difference between healing by primary intention of healing by secondary intention? A. Secondary healing requires surgical debridement for healing to occur. B. Primary healing involves suturing two layers of granulation tissue together C. Presence of more granulation tissue in secondary healing results in scarring D. Healing by secondary intention takes longer because more steps in he healing process are necessary

C. Presence of more granulation tissue in secondary healing results in scarring

To promote effective coughing, deep breathing, and ambulation in the postoperative patient, what is most important for the nurse to do? A. Teach patient controlled breathing B. explain the rationale for these activities C. Provide adequate and regular pain management D. use an incentive spirometer to motivate the patient

C. Provide adequate and regular pain management

Upon administration of a patient PACU nurses about the surgical patient? A. Vital signs B. Surgical Site C. Respiratory Adequacy D. Level of consciousness

C. Respiratory Adequacy

Key interventions for treating soft tissue injury and resulting inflammation are remembered using the acronym RICE. What are the most important actions for the emergency department nurse to do for the patient with an ankle injury? A. Reduce swelling, shine light on wound, control ability, and elicit the history of the injury B. Rub the wound clean, immobilize the area, cover the area protectively, and exercise that leg C. Rest with immobility, apply a cod compress, apply a compress bandage and elevate the ankle D. Rinse the wounded ankle, carry the patient, and extend the ankle with inaging

C. Rest with immobility, apply a cod compress, apply a compress bandage and elevate the ankle

In addition to ambulation, which nursing intervention could be implemented to prevent or treat the postoperative complication of syncope? A. Monitor VS after ambulation B. So not allow the patient to eat before ambulation C. Slowly progress to ambulation with slow changes in position D. Have the patient deep breath and cough before getting out of bed

C. Slowly progress to ambulation with slow changes in position

During the physical assessment of the patient with early to moderate RA, what should the nurse expect to find? A. Hepatomegaly B. Herberden's nodes C. Spindle-shaped fingers D. crepitus on joint movement

C. Spindle-shaped fingers Fingers of patients with moderate RA: - spindle shaped fingers from synovial hypertrophy and thickening of joints - have no joint deformities, but limited mobility - have adjacent muscle atrophy - may be inflammed Hepatomegaly is found with Felty Syndrome with severe nodule forming RA. Herberden's nodes and crepitus is found in OA

Which statement accurately describes vitamin deficiencies? A. The two nutrients most often lacking in the diet of a vegan are vitamin B6 and folic acid B. Vitamin imbalances occur frequently in the US because of impaired absorption C. Surgery on the GI tract may contribute to vitamin deficiencies because of impaired absorption D. Vitamin deficiencies in adults most commonly are clinically manifested by disorders of the skin

C. Surgery on the GI tract may contribute to vitamin deficiencies because of impaired absorption A strict vegan diet= lacks B12 and iron Vitamin imbalances rare in developed countries Vitamin deficiencies in adults are most commonly clinically manifested by neurological disorders

A patient has lateral angulation of the large toe toward the second toe. The nurse knows that treatment will include what? A. Metatarsal arch support B. Trimming with a scalpel after softening C. Surgery to remove the bursal sac and bony enlargement D. Intraarticular corticosteroids and passive manual streaching

C. Surgery to remove the bursal sac and bony enlargement This patient has a hallux valgus (bunion). Metatarsal arch support is a treatment for hammer toe. A corn is trimmed with a scalpel after softening. Intraarticular corticosteroids and passive manual streaching is a treatment for hallux rigidus from osteoarthritis

The patient's wound is not healing, so the health care provider is going to send the patient home with negative pressure would therapy or a "wound vac" device. What will the caregiver need to understand about the use of this device? A. Wound must be cleaned daily B. The patient will be placed in a hyperbaric chamber C. The occlusive dressing must be sealed tightly to the skin D. The diet will not be as important with this sort of treatment.

C. The occlusive dressing must be sealed tightly to the skin

Which nutrients aid in capillary synthesis and collagen production by the fibroblasts in wound healing? A. Fats B. Proteins C. Vitamin C D. Vitamin A

C. Vitamin C

What should be included in the instructions given tot he postoperative patient before discharge? A. Need for follow-up care with home care nurses B. Directions for maintaining routine postoperative diet C. Written information about self-care during recuperation D. Need to restrict all activity until surgical healing is complete

C. Written information about self-care during recuperation

How will an obstruction at the ampulla of Vater affect the digestion of all nutrients? A. Bile is responsible for emulsification of all nutrients and vitamins. B. Intestinal digestive enzymes are released through the ampulla of Vater C. both bile and pancreatic enzymes enter the duodenum at the ampulla of Vater D. Gastric contents can only pass to the duodenum when the ampulla of Vater is open

C. both bile and pancreatic enzymes enter the duodenum at the ampulla of Vater The ampulla of Vater is the site where the pancreatic duct and common bile duct enter the duodenum and the opening and closing of the ampulla is controlled by the sphincter of Oddi.

What is the most important nursing intervention for the prevention and treatment of pressure ulcers? A. Using pressure-reduction devices B. Massaging pressure areas with lotion C. repositioning the patient a minimum of every 2 hours D. Using lift sheets and trapeze bars to facilitate movement

C. repositioning the patient a minimum of every 2 hours

A patient with acute pharyngitis is seen at a clinic with fever and sever throat pain that affects swallowing. On inspection, the throat is reddened and edematous with patchy yellow exudates. The nurse anticipates that collaborative management will include. A. treatment with antibiotics B. treatment with antifungal agents C.a throat culture or rapid strep antigen test D. treatment with medication only if the pharyngitis does not resolve in 3-4 days

C.a throat culture or rapid strep antigen test Although untreated beta-hemolytic streptococcal infections may lead to rheumatic heart disease or glomerulonephritis, antibiotic treatment is not recommended until strep infections are definitely diagnosed with culture or antigen tests.

What is the leading cause of death in the U.S.? a: cancer b: Diabetes mellitus c: coronary artery disease d: cerebrovascular accident e: chronic obstructive pulmonary disorder

C: coronary artery disease

Identify the acid-base balance pH 7.35 PaCO2 60 mm Hg HCO3 40 mEq/L

Compensated metabolic acidosis

Identify the acid-base balance pH 7.44 PaCO2 54 mm Hg HCO3 36 mEq/L

Compensated metabolic alkalosis

List in order the nociceptive pain processes that occur to communicate tissue damage to the CNS. a: perception b: modulation c: transmission d: transduction

D C A B *corrected from previous answer of dcba*

Which actions prevent the dislodgement of a tracheostomy tube in the first 3 days after it's placement? (select all that apply) A. Provide tracheostomy care every 24 hours B. Keep the patient in the semi-fowlers position at all times C. Keep a same size or larger replacement tube at the bedside. D. Tracheostomy ties are not changed for 24 hours after a tracheostomy procedure E. Suction the tracheostomy tube when there is a moist cough or a decreased SpO2 F. A physician performs the first tube change, no sooner than 7 days after the tracheostomy.

D, E, F -Tracheostomy care is done every 8 hours -Semi-fowlers position will not affect trach dislodgment -Keeping a replacement tube at beside can speed replacement if the tube is displace, but will not affect risk of displacement

The nurse realizes that the patient understands the teaching about decreasing the risk for antibiotic-resistant infection when the patient says which of the following? A. " I know I should take the antibiotic for one day after I feel better." B. "i want an antibiotic order for my cold so i can feel better soon." C. "I always save some pills because I get the illness again after I first feel better" D. "I fill follow the directions for taking the antibiotics so I will get over this infection"

D. "I fill follow the directions for taking the antibiotics so I will get over this infection"

Goals for patient safety in the OR include the Universal Protocol. What is included in this protocol? A. All surgical centers of any type must submit reports on patient safety inferactions to the accreditation agencies B. Members of the surgical team stop whatever they are doing to check that all sterile items have been prepared properly. C. Members of the surgical team pause right before surgery to mediate for 1 minute to decrease stress and possible errors. D. A surgical timeout is performed just before the procedure is started to verify patient identity, surgical procedure, and surgical site.

D. A surgical timeout is performed just before the procedure is started to verify patient identity, surgical procedure, and surgical site.

The health care provider has ordered IV morphine q2-4 hr, PRN for a patient following major abdominal surgery. When should the nurse plan to administer the morphine? A. Before all planned painful activities B. Every 2-4 hours during the first 48 hours C. Every 4 hours as the patient requests the medication D. After assessing the nature and intensity of the patient's pain

D. After assessing the nature and intensity of the patient's pain

What is the most common cause of secondary protein-calorie malnutrition in the US? A. The unavailability of foods high in protein B. A lack of knowledge about nutritional needs C. A lock of money to purchase high-protein foods D. An alteration in ingestion, digestion, absorption or metabolism

D. An alteration in ingestion, digestion, absorption or metabolism In the US, there protein intake is high and of good quality, protein-calorie malnutrition most commonly results from problems of the GI system.

A 68-year-old patient is in the office for a physical. She notes that she no longer has regular bowel movements. Which suggestion by the nurse would be most helpful. A. Take an additional laxative to stimulate defecation B. Eat less acidic food to enable GI system to increase peristalsis C. Eat less food at each meal to prevent feces from backing up related to slow peristalsis D. Attempt defecation after breakfast because gastrocolic reflexes increase colon peristalsis

D. Attempt defecation after breakfast because gastrocolic reflexes increase colon peristalsis Gastrocolic and duodenal reflexes are more active for the first meal of the day

What is most likely to cause the pain experienced in the later stages of OA? A. Crepitation B. Bouchard's nodes C. Heberden's nodes D. Bone surfaces rubbing together

D. Bone surfaces rubbing together Crepitation- occurs earlier in disease with loose particles of cartilage in joint cavity Bouchard's nodes & Heberden's nodes- are tender but occur as joint space decreases and as early as 40 years of age

The patient is admitted from home with a stage II pressure ulcer. this wound is classified as a yellow wound using the red-yellow-black concept of wound care. What is the nurse likely to observe when she does her wound assessment? A. Serosanguenous drainage B. Adherent grey necrotic tissue C. Clean, moist granulating tissue D. Creamy ivory to yellow-green exudate

D. Creamy ivory to yellow-green exudate

A patient with a complicated vaginal hysterectomy. The student nurse provided perineal care after the patient has a bowel movement. The student nurse tells the nurse there was a lot of light brown, smelly drainage seeping from the perianal area. What should the nurse suspect when assessing this patient. A. Dehiscence B. Hemorrhage C. Keloid Formation D. Fistula Formation

D. Fistula Formation

During care of patients, what is the most important precaution for preventing transmission of infection? A. Wearing face and eye protection during routine daily care of the patient B. Wearing nonsterile gloves when in contact with body fluids, excretions and contaminated items C. Wearing a gown to protect the skin and clothing during patient care activities likely to soil clothing D. hand washing after touching fluids and secretions and removing gloves, as well as between patient contacts

D. Hand washing after touching fluids and secretions and removing gloves, as well as between patient contacts

What does radicular pain that radiates down the buttock and below the knee, along the distribution of the sciatic nerve, generally indicate? A. Cervical disc herniation B. Acute lumbosacral strain C. Degenerative disc disease D. Herniated intervertebral disc

D. Herniated intervertebral disc Cervical disc herniation- pain radiating into arms and hands Acute lumbosacral strain- acute low back pain Degenerative disc disease- degeneration of disks, normal to aging

What characteristics are common in spondyloarthritides associated with human leukocyte antigen (HLA)-B27? A. Symmetric polyarticular arthristis B. Absence of extraarticular disease C. Presence of rheumatoid factor and antibodies D. High levels of involvement of sacroiliac joints and the axial skeleton

D. High levels of involvement of sacroiliac joints and the axial skeleton

What effect does the action of this compliment system have on inflammation? A. Modifies that inflammatory response to prevent stimulation of pain B. Increases body temperature, resulting in destruction of microorganisms C. Produces prostaglandins and leukotrienes that increase blood flow, edema and pain D. Increases inflammatory responses of vascular permeability, chemotaxis and phagocytosis

D. Increases inflammatory responses of vascular permeability, chemotaxis and phagocytosis

Which nursing interventions for a patient with stage IV sacral pressure ulcer are most appropriate to assign or delegate to a LPN (select all that apply). A. Assess and document wound appearance B. Teach the patient pressure ulcer risk factors C. Choose the type of dressing to apply to the ulcer D. Measure the size (width, length, depth) of the ulcer E. Assist the patient to change positions at frequent intervals

D. Measure the size (width, length, depth) of the ulcer E. Assist the patient to change positions at frequent intervals

To evaluate the effect of nutritional interventions for a patient with protein-calorie malnutrition , what is the best indicator to use? A. Height and body weight B. BMI C. Weight in realtion to ideal body weight D. Mid-upper arm circumference and triceps skin fold

D. Mid-upper arm circumference and triceps skin fold Anthropometric measurements, including these measurements are good indicators of lean body mass and skeletal protein reserves are are valuable in evaluating persons who may have been or are being treated for acute protein malnutrition. The other measurements do not specifically address muscle mass.

Thirty-six hours postoperatively a patient has a terperature of 100 F. What is the most likely cause of this temperature elevation? A. Dehydration B. Wound infection C. Lung congestion and atelectasis D. Normal surgical stress resonse

D. Normal surgical stress resonse

What type of bone tumor is a benign overgrowth of bone and cartilage and may transform into a malignant form? A. Endochroma B. Osteoclastoma C. Ewing's Sarcoma D. Osteochondroma

D. Osteochondroma Endochroma- benign but intramedullary cartilage tumor found in a cavity of a single hand or foot bone Osteoclastoma- benign bone tumor with a high rate of recurrence, but does not become malignant Ewing's Sarcoma- malignant and develops in the medullary cavity of longbones

The nurse receives an evening report on a patient who underwent posterior nasal packing for epitaxis earlier in the day. What is the first patient assessment the nurse should make? A. Patient's temp B. Level of the patient's pain C. Drainage on the nasal dressing D. Oxygen saturation by pulse ox

D. Oxygen saturation by pulse ox

What best describes the manifestations of OA? A. Smaller joints are typically affected first B. There is joint stiffness after periods of inactivity C. Joint stiffness is accompanies by fatigue, anorexia and weight loss D. Pain and immobility may be aggravated by falling barometric pressure.

D. Pain and immobility may be aggravated by falling barometric pressure. -OA affects weight bearing joints of knees and hips -Stiffness occurs on arising, but subsides after 30 min - Joint stiffness is accompanies by fatigue, anorexia and weight loss are signs of RA

The patient will be placed under moderate sedation to allow realignment of a fracture in the emergency department. When the family asks about this anesthesia, what should the nurse tell them? A. Includes inhalation agents B. Induces high levels of sedation C. frequently used for traumatic injuries D. Patients remain responsive and breathe without assistance

D. Patients remain responsive and breathe without assistance

To prevent airway obstruction in the postoperative patient who is unconscious or semiconscious, what will the nurse do? A. encourage deep breathing B. Elevate the head of the bed C. Administer oxygen per mask D. Position the patient in a side-lying position

D. Position the patient in a side-lying position

A patient is seen a the outpatient clinic for a sudden onset of inflammation and severe pain in the great toe. A definitive diagnosis of gouty arthritis is made on the basis of what? A. A family history of gout B. Elevated urine uric acid levels C. elevated serum uric acid levels D. Presence of monosodium urate crystals in synovial fluid

D. Presence of monosodium urate crystals in synovial fluid

After eating, a patient with an inflamed gallbaladder experiences pain caused by contraction of the gallbladder. What is the mechanism responsible for this action? A. Production of bile by the liver B. Production of secretin by the duodenum C. Release of gastrin from the stomach antrum D. Production of cholecystokinin by the duodenum

D. Production of cholecystokinin by the duodenum cholecystokinin is secreted by the duodenal muscoa when fats and amino acids enter the duodenum and stimulate the gallbladder to release bile to emulsify the fats for digestion.

Monitored anesthesia care (MAC) is being considered for a patient undergoing a cervical dilation and endometrial biopsy in the health care clinic. The patient asks the nurse, What is this MAC?" The nurse's response is based on the knowledge that MAC... A. can be administered only by anesthesiologist or nurse anesthetists B. should never be outside the OR because of the risk of serious complications C. is so safe that it can be administered by nurses with direction form health care providers D. Provides maximum flexibility to match the sedation level with patient and procedure needs.

D. Provides maximum flexibility to match the sedation level with patient and procedure needs.

The health care provider orders a pulmonary angiogram for a patient admitted with dyspnea and hemoptysis. For which problem is this test most commonly used as a diagnostic measure? a. Tuberculosis b. Cancer of the lung C. Airway obstruction D. Pulmonary embolism

D. Pulmonary embolism

What is a clinical manifestation of age-related changes in the GI system that the nurse may find in an older patient? A. Gastric hyperacidity B. Intolerance to fatty foods C. Yellowish tinge to the skin D. Reflux of gastric contents into the esophagus

D. Reflux of gastric contents into the esophagus there is decreased tone of the lower esophageal sphincter with aging. There is a decrease in gastric acidity with aging. Jaundice and intolerance to fatty foods indicate liver or gallbladder disease

Which patient has the highest risk for poor nutritional balance related to decreased ingestion? A. Tuberculosis Infection B. Malabsoption syndrome C. Draining decubitus ulcers D. Severe anorexia resulting from radiation therapy

D. Severe anorexia resulting from radiation therapy Infections create a hyper metabolic state that increases nutritional demand. Malabsorption causes loss of nutrients that are ingested Draining decubitous ulcers are examples of disorders that cause loss of protein and hypermetabolic states.

The patient is transferring from another facility with the description of a sore on her sacrum that is deep enough to see the muscle. What stage of pressure ulcer does the nurse expect to see on admission? A. Stage I B. Stage II C. Stage III D. Stage IV

D. Stage IV

In a patient with leukocytosis with a shift to the left, what does the nurse recognize as causing this finding? A. The compliment system has been activated to enhance phagocytosis B. Monocytes are released into the blood in a larger-than-normal amounts C. The response to cellular injury is not adequate to remove damaged tissue and promote healing D. The demand for neutrophils cause the release of immature neutrophils from the bone marrow

D. The demand for neutrophils cause the release of immature neutrophils from the bone marrow

At the end of the surgical procedure, the perioperative nurse evaluates the patient's response to the nursing care delivered during the perioperative period. What reflects a positive outcome related to the patient's physical status? A. The patient's right to privacy is maintained B. The patient's care is consistent with the perioperative plan of care C. The patient receives consistent and comparable care regardless of the setting D. The patient's respiratory function is consistent with or improved from baseline levels established preoperatively.

D. The patient's respiratory function is consistent with or improved from baseline levels established preoperatively.

Lab findings that the nurse would expect to be present in the patient with RA includes: A. Polycythemia B. Increased IgG C. decreased WBC count D. anti-citrullinated protein antibody (ACPA)

D. anti-citrullinated protein antibody (ACPA) RA findings - anemia, not polycythemia - Normal levels if IgG - increased WBC due to inflammation

Which drainage is drained with a Hemovac? A. Bile B. urine C. gastric contents D. wound drainage

D. wound drainage

What are characteristics of a fenestrated tracheostomy tube? A. The cuff passively fills with air B. Cuff pressure monitoring is not required C. It has 2 tubings with one opening just above the cuff D. Patient can speak with an attached air source with the cuff inflated E. Airway obstruction is likely is the exact steps are not followed to produce speech F. Airflow around the tube and through the window allows speech when the cuff is deflated and the plug inserted

E, F This treacheostomy tube has openings on the outer cannula to allow air to pass over the vocal cords to allow speaking. If the steps of using the fenestrated tracheostomy tube are not completed in the correct order, severe respiratory distress may result.

6. A patient is scheduled for surgery requiring general anesthesia at an ambulatory surgical center. The nurse asks him when he ate last. He replies that he had a light breakfast a couple of hours before coming to the surgery center. What should the nurse do first? a. Tell the patient to come back tomorrow, since he ate a meal. b. Proceed with the preoperative checklist, including site identification. c. Notify the anesthesia care provider of when and what the patient last ate. d. Have the patient void before administering any preoperative medications

Notify the anesthesia care provider of when and what the patient last ate.

Indicate whether the following characteristics of parenteral nutrition apply to central parenteral nutrition (CPN) or peripheral parenteral nutrition(PPN) A. limited 20% glucose B. Tonicity of 1600 mOsm/L C. Nutrients an be infused using smaller volumes D. Supplemental oral feedings E. Long-term nutritional support F. Phlebitis more common G. May use PICC

PPN- A. limited 20% glucose CPN- B. Tonicity of 1600 mOsm/L CPN- C. Nutrients an be infused using smaller volumes PPN- D. Supplemental oral feedings CPN- E. Long-term nutritional support PPN- F. Phlebitis more common CPN- G. May use PICC

Identify the acid-base imbalance pH 7.20 PaCO2 25 mm Hg HCO3 15 mEq/L

Partially compensated metabolic acidosis

Identify the acid-base imbalance pH 7.62 PaCO2 48 mm Hg HCO3 45 mEq/L

Partially compensated metabolic alkalosis

List the 5 dimensions of pain

Physiologic: anatomic and physical determinants of pain Affective: emotional response to pain Cognitive: the beliefs, attitudes and meanings attributed to pain Behavioral: observable actions that express or control pain Sociocultural: age, gender, family and caregiver influence, culture that influences pain experience

Indicate whether the following nursing actions must be performed by the RN or if they can be delegated to a UAP A. Inserting NG tube B. Weigh the patient after enteral feedings C. Teach the patient about home gastric tube care D. Remove an NG tube E. Provide oral care to patients with NG tube F. Position patient receiving enteral feedings G. Monitor patient with continuous feedings for complications H. respond to infusion pump alarm by reporting it to an RN or LPN

RN- A. Inserting NG tube UAP- B. Weigh the patient after enteral feedings RN- C. Teach the patient about home gastric tube care RN- D. Remove an NG tube UAP- E. Provide oral care to patients with NG tube UAP- F. Position patient receiving enteral feedings RN- G. Monitor patient with continuous feedings for complications UAP- H. respond to infusion pump alarm by reporting it to an RN or LPN

What are three common factors known to precipitate placement in long-term care facility.

Rapid patient deterioration Caregiver exhaustion Alteration in, or loss of, family support system

Identify the acid-base imbalance pH 7.26 PaCO2 56 mm Hg HCO3 24 mEq/L

Respiratory acidosis

Identify the acid-base imbalance pH 7.50 PaCO2 30 mm Hg HCO3 24 mEq/L

Respiratory alkalosis

What does the acronym SCALES stand for in assessment of nutrition indicators in frail older adults.

S-sadness C- cholesterol (high) A- albumin (low) L- loss (or gain of weight) E- eating problems S- shopping (and food preparation)

A patient experiences increase RBC destruction from a mechanical heart valve prosthesis. Describe what happens to the bilirubin that is released from the breakdown of hemoglobin from the RBC's

The billirubin from the Hgb is insoluble (unconjugated) and attached to albumin in the blood, removed by the liver, combined with glucuronic acid to become soluble (conjugated) and is excreted in bile into the intestine. Bowel bacteria convert some of the bilirubin to urobilinogen. Urobininogen is absorbed into the blood and a small amount of unrobilinogen is excreted by the kidneys in urine, wit the rest being removed by the liver and reexcreted in the bile.

(CH 41) Using the body mass index chart: BMI (kg/m^2)= (weight (lbs)x 703)/ (height(in)^2) a. Determine the BMI for a patient who is 5ft, 5in (164 cm) and weights 202 lbs (91.8 kg). What is the patient's weight classification? b. calculate the waist-to-hip ratio of a woman who has a waist measurement of a 32 in and hip measurement of 36in. What does this value indicate

a, 202x703=142,006/ 4225 (65x65)= 33.6 kg/m^2, Obsesity b. 32/36=0.88, increased risk for health complications

A clinical polysomnography (PSG) may be performed on a patient with signs and symptoms of a sleep disorder. What measures and observations does this study include? a: heart rate monitoring b: noninvasive oxygen saturation c: surface body temperature fluctuations d: blood pressure monitoring (noninvasive) e: airflow measured at the nose and mouth f: muscle tone measured by electromyogram (EMG) g: respiratory effort around the chest and abdomen h: eye movements recorded by electrooculogram (EOG) i: Brain activity recorded by electroencephalogram j: Actigraph watch worn on the wrist to monitor motor activity k: gross body movements monitored via audio and visual recordings

a, b, e, f, g, h, i, k a: heart rate monitoring b: noninvasive oxygen saturation e: airflow measured at the nose and mouth f: muscle tone measured by electromyogram (EMG) g: respiratory effort around the chest and abdomen h: eye movements recorded by electrooculogram (EOG) i: Brain activity recorded by electroencephalogram k: gross body movements monitored via audio and visual recordings

1. A patient with osteomyelitis is treated with surgical debridement with implantation of antibiotic beads. When the patient asks why the beads are used, the nurse answers (select all that apply) a. "The beads are used to directly deliver antibiotics to the site of the infection." b. "There are no effective oral or IV antibiotics to treat most cases of bone infection." c. "This is the safest method of delivering long-term antibiotic therapy for a bone infection." d. "The beads are an adjunct to debridement and oral and IV antibiotics for deep infections." e. "The ischemia and bone death that occur with osteomyelitis are impenetrable to IV antibiotics."

a. "The beads are used to directly deliver antibiotics to the site of the infection." d. "The beads are an adjunct to debridement and oral and IV antibiotics for deep infections."

1. The percentage of daily calories for a healthy individual consists of a. 50% carbohydrates, 25% protein, 25% fat, and <10% of fat from saturated fatty acids. b. 65% carbohydrates, 25% protein, 25% fat, and >10% of fat from saturated fatty acids. c. 50% carbohydrates, 40% protein, 10% fat, and <10% of fat from saturated fatty acids. d. 40% carbohydrates, 30% protein, 30% fat, and >10% of fat from saturated fatty acids.

a. 50% carbohydrates, 25% protein, 25% fat, and <10% of fat from saturated fatty acids.

6. When planning health care teaching to prevent or detect early head and neck cancer, which people would be the priority to target (select all that apply)? a. 65-year-old man who has used chewing tobacco most of his life b. 45-year-old rancher who uses snuff to stay awake while driving his herds of cattle c. 78-year-old woman who has been drinking hard liquor since her husband died 15 years ago d. 21-year-old college student who drinks beer on weekends with his fraternity brothers e. 22-year-old woman who has been diagnosed with human papilloma virus (HPV) of the cervix

a. 65-year-old man who has used chewing tobacco most of his life b. 45-year-old rancher who uses snuff to stay awake while driving his herds of cattle c. 78-year-old woman who has been drinking hard liquor since her husband died 15 years ago e. 22-year-old woman who has been diagnosed with human papilloma virus (HPV) of the cervix

3. Which individuals would be at high risk for low back pain (select all that apply)? a. A 63-year-old man who is a long-distance truck driver b. A 36-year-old 6 ft, 2 in construction worker who weighs 260 lb c. A 28-year-old female yoga instructor who is 5 ft, 6 in and weighs 130 lb d. A 30-year-old male nurse who works on an orthopedic unit and smokes e. A 44-year-old female chef with prior compression fracture of the spine

a. A 63-year-old man who is a long-distance truck driver b. A 36-year-old 6 ft, 2 in construction worker who weighs 260 lb d. A 30-year-old male nurse who works on an orthopedic unit and smokes e. A 44-year-old female chef with prior compression fracture of the spine

4. When teaching a patient about the most important respiratory defense mechanism distal to the respiratory bronchioles, which topic would the nurse discuss? a. Alveolar macrophages b. Impaction of particles c. Reflex bronchoconstriction d. Mucociliary clearance mechanism

a. Alveolar macrophages

A patient who is being admitted to the surgical unit for a hysterectomy paces the floor, repeatedly saying, "I just want this over." What should the nurse do to promote a positive surgical outcome for the patient? a. Ask the patient what her specific concerns are about the surgery. b. Reassure the patient that the surgery will be over soon and she will be fine. c. Redirect the patient's attention to the necessary preoperative preparations. d. Tell the patient she should not be so anxious because she is having a common, safe surgery.

a. Ask the patient what her specific concerns are about the surgery.

What are the characteristics of the carbonic acid - bicarbonate buffer system? (select all that apply) a. CO2 is eliminated by the lung b. Neutralizes HCl acid to yield carbonic acid and salt c. HCO3 formed by neutralization dissociates into H2O and CO2 d. Shifts H+ in and out of cell in exchange for other cations such as potassium and sodium e. Free basic radicals dissociated into ammonia and OH- that combines with H+ to form water

a. CO2 is eliminated by the lung b. Neutralizes HCl acid to yield carbonic acid and salt c. HCO3 formed by neutralization dissociates into H2O and CO2

3e. It is especially important for the nurse to assess for which clinical manifestation(s) in a patient who has just undergone a total thyroidectomy (select all that apply)? a. Confusion b. Weight gain c. Depressed reflexes d. Circumoral numbness e. Positive Chvostek's sign

a. Confusion d. Circumoral numbness e. Positive Chvostek's sign

A patient is scheduled to have a tunneled catheter placed for administration of chemotherapy for breast cancer. When preparing the patient for the catheter insertion, what does the nurse explain about this method of chemotherapy administration? a. Decreases the risk for extravasation at the infusion site. b. Reduces the incidence of systemic side effects of the drug. c. Does not become occluded as peripherally inserted catheters can. d. Allows continuous infusion of the drug directly to the area of the tumor.

a. Decreases the risk for extravasation at the infusion site.

To provide free water and intracellular fluid hydration for a patient with acute gastroenteritis who is NPO, the nurse would expect administration of which infusion? a. Dextrose 5% in water b. Dextrose 10% in water c. Lactated Ringer's solution d. Dextrose 5% in normal saline (0.9%)

a. Dextrose 5% in water

The nurse is preparing a patient for transport to the operating room. The patient is scheduled for a right knee arthroscopy. What actions should the nurse take at this time? (select all that apply) a. Ensure that the patient has voided. b. Verify that the informed consent is signed. c. Complete preoperative nursing documentation. d. Verify that the right knee is marked with indelible marker. e. Ensure that the H&P, diagnosis reports and vital signs are on the chart.

a. Ensure that the patient has voided. b. Verify that the informed consent is signed. c. Complete preoperative nursing documentation. d. Verify that the right knee is marked with indelible marker. e. Ensure that the H&P, diagnosis reports and vital signs are on the chart.

Many herbal products that are commonly taken cause surgical problems. Which herbs listed below should the nurse teach the patient to avoid before surgery to prevent an increase in bleeding for the surgical patient? (select all that apply) a. Garlic b. Fish oil c. Valerian d. Vitamin E e. Astragalus f. Ginkgo biloba

a. Garlic b. Fish oil

Many herbal products that are commonly taken cause surgical problems. Which herbs listed below should the nurse teach the patient to avoid before surgery to prevent an increase in bleeding for the surgical patient? (select all that apply) a. Garlic b. Fish oil c. Valerian d. Vitamin E e. Astragalus f. Ginkgo biloba

a. Garlic b. Fish oil d. Vitamin E f. Ginkgo biloba

What type of procedural information should be given to a patient in preparation for ambulatory surgery? (select all that apply) a. How pain will be controlled b. Any fluid and food restrictions c. Characteristics of monitoring equipment d. What odors and sensations may be experienced. e. Technique and practice of coughing and deep breathing, if appropriate.

a. How pain will be controlled b. Any fluid and food restrictions e. Technique and practice of coughing and deep breathing, if appropriate.

Which electrolyte imbalance is associated with the following cause? (may be one or more) Diabetes insipidus _______________ a. Hypernatremia b. Hyponatremia c. Hyperkalemia d. Hypokalemia e. Hypercalcemia f. Hypocalcemia g. Hyperphosphatemia h. Hypophosphatemia i. Hypermagnesemia j. Hypomagnesemia

a. Hypernatremia

Which electrolyte imbalance is associated with the following cause? (may be one or more) Osmotic diuresis _______________ a. Hypernatremia b. Hyponatremia c. Hyperkalemia d. Hypokalemia e. Hypercalcemia f. Hypocalcemia g. Hyperphosphatemia h. Hypophosphatemia i. Hypermagnesemia j. Hypomagnesemia

a. Hypernatremia

Which electrolyte imbalance is associated with the following cause? (may be one or more) Hyperaldosteronism ________ a. Hypernatremia b. Hyponatremia c. Hyperkalemia d. Hypokalemia e. Hypercalcemia f. Hypocalcemia g. Hyperphosphatemia h. Hypophosphatemia i. Hypermagnesemia j. Hypomagnesemia

a. Hypernatremia d. Hypokalemia

7. Which of the following criteria must be met for a diagnosis of metabolic syndrome (select all that apply)? a. Hypertension b. Elevated triglycerides c. Elevated plasma glucose d. Increased waist circumference e. Decreased low-density lipoproteins

a. Hypertension b. Elevated triglycerides c. Elevated plasma glucose d. Increased waist circumference

7. An 85-year-old patient is assessed to have a score of 16 on the Braden Scale. Based on this information, how should the nurse plan for this patient's care? a. Implement a q2hr turning schedule with skin assessment. b. Place DuoDerm on the patient's sacrum to prevent breakdown. c. Elevate the head of bed to 90 degrees when the patient is supine. d. Continue with weekly skin assessments with no special precautions.

a. Implement a q2hr turning schedule with skin assessment.

3. A 59-year-old man is scheduled for a herniorrhaphy in 2 days. During the preoperative evaluation he reports that he takes ginkgo daily. What is the priority intervention? a. Inform the surgeon, since the procedure may need to be rescheduled. b. Notify the anesthesia care provider, since this herb interferes with anesthetics. c. Ask the patient if he has noticed any side effects from taking this herbal supplement. d. Tell the patient to continue to take the herbal supplement up to the day before surgery.

a. Inform the surgeon, since the procedure may need to be rescheduled.

The nurse is admitting a patient to the critical unit from surgery. Being alert to potential fluid volume alterations, what assessment data will be important for the nurse to monitor to identify early changes in the patient's postoperative fluid volume? (select all that apply) a. Intake and output b. Skin turgor c. Lung sounds d. Respiratory rate e. Level of consciousness

a. Intake and output b. Skin turgor c. Lung sounds d. Respiratory rate e. Level of consciousness

A patient is scheduled for a hemorrhoidectomy at an ambulatory day-surgery center. An advantage of performing surgery at an ambulatory center is a decreased need for a. Laboratory tests an perioperative medications. b. preoperative and postoperative medications. c. psychologic support to alleviate fears of pain and discomfort. d. preoperative nursing assessment related to possible risks and complications.

a. Laboratory tests an perioperative medications.

What are the characteristics of the phosphate buffer system? (select all that apply) a. Neutralizes a strong base to a weak base and water b. Resultant sodium biphosphonate is eliminated by the kidneys c. Free radicals dissociate into H+ and CO2 buffering excess base d. Neutralizes a strong acid to yield sodium biphosphonate, a weak acid, and salt e. Shifts chloride in and out of red blood cells in exchange for sodium bicarbonate, buffering both acids and bases

a. Neutralizes a strong base to a weak base and water b. Resultant sodium biphosphonate is eliminated by the kidneys d. Neutralizes a strong acid to yield sodium biphosphonate, a weak acid, and salt

During the preoperative physical examination, the nurse is alerted to the possibility of compromised respiratory function during or after the surgery in a patient with which problem? a. Obesity b. Dehydration c. Enlarged liver d. Decreased peripheral pulses

a. Obesity

3. After admission of the postoperative patient to the clinical unit, which assessment data require the most immediate attention? a. Oxygen saturation of 85% b. Respiratory rate of 13/min c. Temperature of 100.4°F (38°C) d. Blood pressure of 90/60 mm Hg

a. Oxygen saturation of 85%

7. Teach the patient with fibromyalgia the importance of limiting intake of which foods (select all that apply)? a. Sugar b. Alcohol c. Caffeine d. Red meat e. Root vegetables

a. Sugar b. Alcohol c. Caffeine

A patient is scheduled for hip replacement surgery in the early afternoon, is NPO but receives and ingests a breakfast tray with clear liquids on the morning of surgery. What response does the nurse expect when the anesthesia car provider is notified? a. Surgery will be done as scheduled. b. Surgery will be rescheduled for the following day. c. Surgery will be postponed for 8 hours after the fluid intake. d. A nasogastric tube will be inserted to remove liquids from the stomach.

a. Surgery will be done as scheduled.

Which statements about fluid in the human body are true? (select all that apply) a. The primary hypothalamic mechanism of water intake is thirst. b. Third spacing refers to the abnormal movement of fluid into the interstitial spaces. c. A cell surrounded by hypoosmolar fluid will shrink and die as water moves out of the cell. d. A cell surrounded by hyperosmolar fluid will shrink and die as water moves out of the cell. e. Concentrations of Na+ and K+ in interstitial and intracellular fluids are maintained by the sodium-potassium pump.

a. The primary hypothalamic mechanism of water intake is thirst. d. A cell surrounded by hyperosmolar fluid will shrink and die as water moves out of the cell. e. Concentrations of Na+ and K+ in interstitial and intracellular fluids are maintained by the sodium-potassium pump.

11. The nurse is preparing the patient for a diagnostic procedure to remove pleural fluid for analysis. The nurse would prepare the patient for which test? a. Thoracentesis b. Bronchoscopy c. Pulmonary angiography d. Sputum culture and sensitivity

a. Thoracentesis

3. The nurse is caring for a patient undergoing surgery for a knee replacement. What is critical to the patient's safety during the procedure (select all that apply)? a. Universal protocol is followed. b. The ACP is an anesthesiologist. c. The patient has adequate health insurance. d. The circulating nurse is a registered nurse. e. The patient's allergies are conveyed to the surgical team.

a. Universal protocol is followed. e. The patient's allergies are conveyed to the surgical team.

2. Which antibiotic-resistant organisms cannot be killed by normal hand soap? a. Vancomycin-resistant enterococci b. Methicillin-resistant Staphylococcus aureus c. Penicillin-resistant Streptococcus pneumoniae d. β-Lactamase-producing Klebsiella pneumoniae

a. Vancomycin-resistant enterococci

4. A 17-year-old patient with a leg fracture is scheduled for surgery. She reports that she is living with a friend and is an emancipated minor. She has a statement from the court for verification. Which intervention is most appropriate? a. Witness the permit after consent is obtained by the surgeon. b. Call a parent or legal guardian to sign the permit, since the patient is under 18. c. Obtain verbal consent, since written consent is not necessary for emancipated minors. d. Investigate your state's nurse practice act related to emancipated minors and informed consent.

a. Witness the permit after consent is obtained by the surgeon.

The nurse asks a preoperative patient to sign a surgical consent form as specified by the surgeon and then signs the form after the patient does so. By this action, what is the nurse doing? a. Witnessing the patient's signature. b. Obtaining informed consent from the patient for the surgery. c. Verifying that the consent for surgery is truly voluntary and informed. d. Ensuring that the patient is mentally competent to sign the consent form.

a. Witnessing the patient's signature.

5. A student nurse asks the RN what can be measured by arterial blood gases (ABGs). The RN tells the student that the ABGs can measure (select all that apply) a. acid-base balance. b. oxygenation status. c. acidity of the blood. d. glucose bound to hemoglobin. e. bicarbonate (HCO3 − ) in arterial blood.

a. acid-base balance. b. oxygenation status. c. acidity of the blood. e. bicarbonate (HCO3 − ) in arterial blood.

An appropriate care choice for an older adult who lives with an employed daughter but requires help with activities of daily living is a. adult day care. b. long-term care. c. a retirement center. d. an assisted living facility

a. adult day care.

What should the nurse do when preparing a patient for a pulmonary angiogram scan? a. assess the patient for iodine allergy b. implement npo orders for 6-12 hours before the test c. explain the test before the patient signs the informed consent form d. inform the patient that radiation isolation for 24 hours after the test is necessary

a. assess the patient for iodine allergy

(CH 41) What information should be included in the dietary teaching for the patient following a Roux-en-Y gastric bypass a. avoid sugary foods and limit fluids to prevent dumping syndrome b. gradually increase the amount of food ingested to preoperative levels c maintain a long-term liquid diet to prevent damage to surgical site consume foods high in complex carbohydrates, protein, and fibers to add bulk to contents

a. avoid sugary foods and limit fluids to prevent dumping syndrome

5. The increased risk for falls in the older adult is most likely due to a. changes in balance. b. decrease in bone mass. c. loss of ligament elasticity. d. erosion of articular cartilage.

a. changes in balance.

6. A patient with a fracture of the pelvis should be monitored for a. changes in urine output. b. petechiae on the abdomen. c. a palpable lump in the buttock. d. sudden increase in blood pressure.

a. changes in urine output.

3. Health risks associated with obesity include (select all that apply) a. colorectal cancer. b. rheumatoid arthritis. c. polycystic ovary syndrome. d. nonalcoholic steatohepatitis. e. systemic lupus erythematosus.

a. colorectal cancer. c. polycystic ovary syndrome. d. nonalcoholic steatohepatitis.

4. A patient with tendonitis asks what the tendon does. The nurse's response is based on the knowledge that tendons a. connect bone to muscle. b. provide strength to muscle. c. lubricate joints with synovial fluid. d. relieve friction between moving parts.

a. connect bone to muscle.

4. The best method for determining the risk of aspiration in a patient with a tracheostomy is to a. consult a speech therapist for swallowing assessment. b. have the patient drink plain water and assess for coughing. c. assess for change of sputum color 48 hours after patient drinks small amount of blue dye. d. suction above the cuff after the patient eats or drinks to determine presence of food in trachea.

a. consult a speech therapist for swallowing assessment.

(CH 23) What is the most common diagnostic test used to determine a causative agent of the skin infections? a. culture b. tzanck test c. immunofluorescent studies d. potassium hydroxide (KOH) slides

a. culture

4. A patient is jaundiced and her stools are clay colored (gray). This is most likely related to a. decreased bile flow into the intestine. b. increased production of urobilinogen. c. increased production of cholecystokinin. d. increased bile and bilirubin in the blood.

a. decreased bile flow into the intestine.

4. The nurse's primary responsibility for the care of the patient undergoing surgery is a. developing an individualized plan of nursing care for the patient. b. carrying out specific tasks related to surgical policies and procedures. c. ensuring that the patient has been assessed for safe administration of anesthesia. d. performing a preoperative history and physical assessment to identify patient needs.

a. developing an individualized plan of nursing care for the patient.

a 75-year old patient who is breathing room air has the following arterial blood gas (ABG) results: ph. 7.40 PaO2 74 mm HG, SaO2 92%, PaCO2 40 mm Hg. What is the most appropriate action by the nurse? a. document results b. repeat within an hour to validate findings c. encourage deep breathing and coughing to open alveoli d. initiate pulse oximetry for continuous monitoring of the patient's oxygen status

a. document the results A normal finding in ABGs in the older adult include a small decrease in PaO2 and SaO2, but normal pH and PaCO2 Normal pH: 7.35-7.45 Normal PaCO2: 35-45 mnHg

8. When assessing activity-exercise patterns related to respiratory health, the nurse inquires about a. dyspnea during rest or exercise. b. recent weight loss or weight gain. c. ability to sleep through the entire night. d. willingness to wear oxygen equipment in public.

a. dyspnea during rest or exercise.

2. While performing passive range of motion for a patient, the nurse puts the ankle joint through the movements of (select all that apply) a. flexion and extension. b. inversion and eversion. c. pronation and supination d. flexion, extension, abduction, and adduction. e. pronation, supination, rotation, and circumduction

a. flexion and extension. b. inversion and eversion.

3b. The nursing care for a patient with hyponatremia includes a. fluid restriction. b. administration of hypotonic IV fluids. c. administration of a cation-exchange resin. d. increased water intake for patients on nasogastric suction.

a. fluid restriction.

2. The nurse explains to a patient with a fracture of the distal shaft of the humerus who is returning for a 4-week checkup that healing is indicated by a. formation of callus. b. complete bony union. c. hematoma at fracture site. d. presence of granulation tissue.

a. formation of callus.

(CH 41) A patient asks the nurse about taking phentermine and topiramate (Qsymia) for weight loss. to avoid side effects, it is important for the nurse to determine whether the patient has a history of a. glaucoma b. hypertension c. valvular heart disease d. irritable bowel disease

a. glaucoma

7. During the postoperative period, the nurse instructs the patient with an above-the-knee amputation that the residual limb should not be routinely elevated because this position promotes a. hip flexion contractures. b. skin irritation and breakdown. c. clot formation at the incision site. d. increased risk of wound dehiscence.

a. hip flexion contractures.

6. When positioning a patient in preparation for surgery, the nurse understands that injury to the patient is most likely to occur as a result of a. incorrect musculoskeletal alignment. b. loss of perception of pain or pressure. c. pooling of blood in peripheral vessels. d. disregarding the patient's need for modesty.

a. incorrect musculoskeletal alignment.

3. Insufficient sleep is associated with (select all that apply) a. increased body mass index. b. increased insulin resistance. c. impaired cognitive functioning. d. increased immune responsiveness. e. increased daytime body temperature

a. increased body mass index. b. increased insulin resistance. c. impaired cognitive functioning.

4. The lungs act as an acid-base buffer by a. increasing respiratory rate and depth when CO2 levels in the blood are high, reducing acid load. b. increasing respiratory rate and depth when CO2 levels in the blood are low, reducing base load. c. decreasing respiratory rate and depth when CO2 levels in the blood are high, reducing acid load. d. decreasing respiratory rate and depth when CO2 levels in the blood are low, increasing acid load.

a. increasing respiratory rate and depth when CO2 levels in the blood are high, reducing acid load.

2. The nurse recognizes that a patient with newly diagnosed breast cancer is using an emotion-focused coping process when she a. joins a support group for women with breast cancer. b. considers the pros and cons of the various treatment options. c. delays treatment until her family can take a weekend trip together. d. tells the nurse that she has a good prognosis because the tumor is small.

a. joins a support group for women with breast cancer.

7. Individuals with dark skin are more likely to develop a. keloids. b. wrinkles. c. skin rashes. d. skin cancer.

a. keloids.

(CH 41) Normally, which hormones and peptides affect appetite (select all that apply) a. leptin b. insulin c. ghrelin d. peptide YY e. neuropeptide Y f. cholecystokinin

a. leptin b. insulin c. ghrelin d. peptide YY e. neuropeptide Y f. cholecystokinin

(CH 41) The nurse is teaching a moderately obese woman interventions fro the management of obesity. Initially, which strategies will support restriction dietary intake to below energy requirements (select all that apply) a. limit alcohol b. rest when fatigued c. determine portion sizes d. 1800-2200 calorie diet e. attend overeaters anonymous

a. limit alcohol c. determine portion sizes

6. A patient with sleep apnea would like to avoid using a nasal CPAP device if possible. To help him reach this goal, the nurse suggests that the patient a. lose excess weight. b. take a nap during the day. c. eat a high-protein snack at bedtime. d. use mild sedatives or alcohol at bedtime.

a. lose excess weight.

8. A normal assessment finding of the musculoskeletal system is a. no deformity or crepitation. b. muscle and bone strength of 4. c. ulnar deviation and subluxation. d. angulation of bone toward midline.

a. no deformity or crepitation.

8. A normal physical assessment finding of the GI system is/are (select all that apply) a. nonpalpable liver and spleen. b. borborygmi in upper right quadrant. c. tympany on percussion of the abdomen. d. liver edge 2 to 4 cm below the costal margin. e. finding of a firm, nodular edge on the rectal examination.

a. nonpalpable liver and spleen. c. tympany on percussion of the abdomen.

2. During administration of a hypertonic IV solution, the mechanism involved in equalizing the fluid concentration between ECF and the cells is a. osmosis. b. diffusion. c. active transport. d. facilitated diffusion.

a. osmosis.

6. To assess the skin for temperature and moisture, the most appropriate technique for the nurse to use is a. palpation. b. inspection. c. percussion. d. Auscultation.

a. palpation.

(CH 41) before selecting a weight reduction plan with an obese patient, what is most important for the nurse to first assess? a. patient's motivation to lose weight b. length of time that the patient has been obese c. whether financial consideration will affect the patient's choices d. the patient's anthropometric measures of height, weight, BMI, waist-to-hip ratio, and skinfold thickness.

a. patient's motivation to lose weight

(CH 41) A patient has been on a 1000 calorie diet with a daily exercise routine. in 2 months, the patient has lost 20 lbs (9 kg) toward a goal of 50 lb (23 kg) but is now discouraged that no weight has been lost in the last 2 weeks. what should the nurse tell the patient about this? a. plateau where no weight is lost normally occur during a weight loss program b. a weight considered by the body to be most efficient for functioning has been reached c. A return to former eating habits is the most common cause of not continuing to lose weight d. a steady weight may be due to water gain from eating foods high in sodium

a. plateau where no weight is lost normally occur during a weight loss program

4. The nurse should teach the patient with ankylosing spondylitis the importance of a. regularly exercising and maintaining proper posture. b. avoiding extremes in environmental temperatures. c. maintaining usual physical activity during flare-ups. d. applying hot and cool compresses for relief of local symptoms.

a. regularly exercising and maintaining proper posture.

6. Before discharge from the same-day surgery unit, instruct the patient who has had a surgical correction of bilateral hallux valgus to a. rest frequently with the feet elevated. b. soak the feet in warm water several times a day. c. expect the feet to be numb for the next few days. d. expect continued pain in the feet, since this is not uncommon.

a. rest frequently with the feet elevated.

1. Emerging infections can affect health care by (select all that apply) a. revealing antibiotic resistance. b. generating scientific discoveries. c. creating a strain on limited resources. d. challenging established medical traditions. e. limiting travel options for nursing personnel.

a. revealing antibiotic resistance. b. generating scientific discoveries. c. creating a strain on limited resources. d. challenging established medical traditions. e. limiting travel options for nursing personnel.

(CH 41) Which teaching points are important when providing information to a patient with metabolic syndrome (select all that apply) a. stop smoking b. monitor weight daily c. increase level of activity d. decrease saturated fat intake e. reduce weight and maintain lower weight f. check blood glucose each morning prior to eating

a. stop smoking c. increase level of activity d. decrease saturated fat intake e. reduce weight and maintain lower weight

1. An overweight patient (BMI 28.1 kg/m2 ) is scheduled for a laparoscopic cholecystectomy at an outpatient surgery setting. The nurse knows that a. surgery will involve multiple small incisions. b. this setting is not appropriate for this procedure. c. surgery will involve removing a portion of the liver. d. the patient will need special preparation because of obesity.

a. surgery will involve multiple small incisions.

1. To promote the release of surfactant, the nurse encourages the patient to a. take deep breaths. b. cough five times per hour to prevent alveolar collapse. c. decrease fluid intake to reduce fluid accumulation in the alveoli. d. sit with head of bed elevated to promote air movement through the pores of Kohn.

a. take deep breaths.

After which test should the nurse observe the patient for symptoms of a pneumothorax? a. thoracentesis b. pulmonary function test c. ventilation-perfusion scan d. positron emission tomography PET scan.

a. thoracentesis

(CH 23) A women calls the health clinic and describes a rash that she has over the abdomen and chest. she tells the nurse it has raised, fluid-filled, small blisters that are distinct. a. Identify the type of primary skin lesion described by this patient b. what is the distribution terminology for these lesions? c. What additional information does the nurse need to document the critical components of these lesions?

a. vesicles b. discrete, localized to the chest and abdomen c. color, size, and configuration

3a. An older woman was admitted to the medical unit with dehydration. Clinical indications of this problem are (select all that apply) a. weight loss. b. dry oral mucosa. c. full bounding pulse. d. engorged neck veins. e. decreased central venous pressure.

a. weight loss. b. dry oral mucosa. e. decreased central venous pressure.

5. The nurse assessed the patient's skin lesions as firm, edematous, irregularly shaped with a variable diameter. They would be called a. wheals. b. papules. c. pustules. d. plaques.

a. wheals.

4. The nurse confirms initial placement of a blindly inserted smallbore NG feeding tube by a. x-ray. b. air insufflation. c. observing patient for coughing. d. pH measurement of gastric aspirate.

a. x-ray.

When is a fat embolism most likely to occur? a: 24-48 hours following a factured tibia b: 36-72 hours following a skull fracture c: 4-5 days following a fractured femur d: 5-6 days following a pelvic fracture

a: 24-48 hours following a factured tibia

The patient has chronic pain that is no longer relieved with oral morphine. Which medication would the nurse expect to be ordered to provide better pain relief for this patient? a: Duragesic b: oramorph SR c: hydrocodone d: intranasal butorphanol (stadol)

a: Duragesic

A patient who has been taking ibuprofen (motrin) and imipramine (tofranil) for control of cancer pain is having increased pain. What would the health care provider recommend as an appropriate change in the medication plan? a: add PO oxycodone to the other medications b: substitute PO propoxyphene, a mild opioid, for imipramine c: add transdermal fentanyl to the use of the other medications d: substitute a PO hydrocodone with acetaminophen for the other medications

a: add PO oxycodone to the other medications

What emergency considerations must be included with facial fractures? a: airway patency b: oral examination c: cervical spine injury d: cranial nerve assessment e: immobilization of the jaw

a: airway patency c: cervical spine injury

What is the function of a tendon? a: attaches the muscle to the bone b: connects bone to bone at the joint c: connects cartilage to muscle in joints d: attaches synovium to the joint capsule

a: attaches the muscle to the bone

Which statements about older people are only myths and illustrate the concept of ageism? select all that apply a: can't teach an old dog new tricks b: old people are not sexually active c: most old people live independently d: most older adults can no longer synthesize new information e: most older adults lose interest in life and wish they would die

a: can't teach an old dog new tricks b: old people are not sexually active d: most older adults can no longer synthesize new information e: most older adults lose interest in life and wish they would die

When obtaining information about the patient's use of medications, the nurse recognizes that both bone and muscle function may be impaired when the patient reports taking what type of drug? a: corticosteroids b: oral hypoglycemic agents c: potassium-depleting diuretics d: NSAIDS

a: corticosteroids

What surgical treatment will the nurse prepare the patient for in the presence of compartment syndrome? a: fasciotomy b: amputation c: internal fixation d: release of tendons

a: fasciotomy

A patient with a fractured right hip has an anterior open reduction and internal fixation of the fracture. What should the nurse plan to do postoperatively? a: get the patient up to the chair on the first postoperative day b:position the patient only on the back and the unoperative side c: keep the leg abductor pillow on the patient even when bathing d: ambulate the patient with partial weight bearing by discharge

a: get the patient up to the chair on the first postoperative day

In performing range of motion (ROM) with a patient, the nurse puts each joint through its full movement. Which joints are capable of abduction and adduction? a: hip b: knee c: wrist d: elbow e: thumb f: shoulder

a: hip c: wrist e: thumb f: shoulder

What is a disadvantage of open reduction and internal fixation of a fracture compared to a closed reduction? a: infection b: skin irritation c: nerve impairment d: complications of immobility

a: infection

A patient who had a below the knee amputation is to be fitted with a temporary prosthesis . It is most important for the nurse to teach the patient to do what? a: inspect the residual limb daily for irritation b: apply an elastic shrinker before applying the prosthesis c: perform ROM exercises to the affected leg four times a day d: apply alcohol to the residual limb every morning and evening to toughen skin

a: inspect the residual limb daily for irritation

Which kind of hip fracture is usually repaired with hip prosthesis? a: intracapsular b: extracapsular c: subtrochanteric d: intertrochanteric

a: intracapsular

A patient is receiving a continuous infusion of morphine via an epidural catheter following a major abdominal surgery. Which actions should the nurse include in the plan of care? Select all that apply a: label the catheter as an epidural access b: assess the patient's pain relief frequently c: use sterile technique when caring for the catheter d: monitor the patient's Level of Consciousness e: monitor patient vital signs (bp, hr, resp) f: assess the motor and sensory function of the patient's lower extremities

a: label the catheter as an epidural access b: assess the patient's pain relief frequently c: use sterile technique when caring for the catheter d: monitor the patient's Level of Consciousness e: monitor patient vital signs (bp, hr, resp) f: assess the motor and sensory function of the patient's lower extremities

An immediate prosthetic fitting during surgery is used for a patient with a traumatic below-the-knee amputation. During the immediate postoperative period, what is a priority nursing intervention? a: monitor the patient's vital signs b: assess the incision for hemorrhage c: elevate the residual limb on pillows d: have the patient flex and extend the knee every hour

a: monitor the patient's vital signs

A patient with colorectal cancer has continuous, poorly localized abdominal pain at an intensity of 5 on a scale of 0-10. How does the nurse teach the patient to use pain medications? a: on an around-the-clock schedule b: as often as necessary to keep the pain controlled c: by alternating two different types of drugs to prevent tolerance d: when the pain cannot be controlled with distraction or relaxation

a: on an around-the-clock schedule

The patient had a fracture. At 3 weeks to 6 months there is a clinical union, and this is the first stage of healing that is sufficient to prevent movement of the fracture site when the bones are gently stressed. How is this stage of fracture healing documented. a: ossification b: remodeling c: consolidation d: callus formation

a: ossification

A patient comes to the clinic with a complaint of dull pain in the anterior and posterior neck. On examination, the nurse notes that the patient has full ROM of the neck and no throat redness or enlarged head or neck lymph nodes. What will be the nurse's next appropriate assessment indicated by these findings? a: palpation of the liver b: auscultation of bowel sounds c: inspection of the patient's ears d: palpation for the presence of left flank pain

a: palpation of the liver

Which nursing actions would demonstrate the nurse's understanding of the concept of providing safe care without using restraints? (select all that apply) a: placing patients with fall risk in low beds b: asking simple yes-no questions to clarify patient's needs c: making hourly rounds on patients to assess for pain and toileting needs d: placing a distruptive patient near the nurses' station in a chair with a seat belt e: applying a jacket vest loosely so the patient can turn but cannot climb out of bed.

a: placing patients with fall risk in low beds b: asking simple yes-no questions to clarify patient's needs c: making hourly rounds on patients to assess for pain and toileting needs

What is a mental status assessment of the older adult especially important in determining? a: potential for independent living b: eligibility for federal health programs c: service and placement needs of the individual d: whether the person should be classified as frail.

a: potential for independent living

When teaching a 69 Y.O. patient about self-care, what will promote health (select all that apply)? a: proper diet b: immunizations c: teaching chair yoga d: demostrating balancing techniques e: participation in health promotion activities

a: proper diet b: immunizations c: teaching chair yoga d: demostrating balancing techniques e: participation in health promotion activities

The x-ray shows that the patient's fracture is at the remodeling stage. What characteristics of the fracture healing processes are happening at this stage (select all that apply). a: radiologic union b: absorption of excess cells c: return to pre-injury strength and shape d: semisolid blood clot at the ends of fragments e: deposition and absorption of bone in response to stress f: unorganized network of bone woven around fracture parts

a: radiologic union b: absorption of excess cells c: return to pre-injury strength and shape e: deposition and absorption of bone in response to stress

What strategies could decrease the distress of rotating shifts for nurses? select all that apply a: take a brief onsite nap b: use sleep hygiene practices c: sleep just before going to work d: maintain consistent sleep/wake schedules even on days off (if possible) Negotiate to control work schedule rather than having someone else impose the schedule

a: take a brief onsite nap b: use sleep hygiene practices c: sleep just before going to work d: maintain consistent sleep/wake schedules even on days off (if possible) Negotiate to control work schedule rather than having someone else impose the schedule

A patient with a fractured femur experiences the complications of malunion. The nurse recognizes that what happens with this complication? a: the fracture heals in an unsatisfactory position b: the fracture fails to heal properly despite treatment c: fracture healing progresses more slowly than expected d: loss of bone substances occurs as a result of immobilization.

a: the fracture heals in an unsatisfactory position

Once generated, what may block the transmission of an action potential along a peripheral nerve fiber to the dorsal root of the spinal cord? a: the transmission may be interrupted by such drugs as local anesthetics b: nothing can stop the action potential along an intact nerve until it reaches the spinal cord c: the action potential must cross several synapses, points at which the impulse may be blocked by drugs d: the nerve fiber produces neurotransmitters that may activate nearby nerve fibers to transmit pain impulses.

a: the transmission may be interrupted by such drugs as local anesthetics

Why does the nurse position a patient with an above-the-knee amputation with a delayed prosthetic fitting prone several times a day? a: to prevent flexion contractures b: to assess the posterior skin flap c: debridement d: synovectomy

a: to prevent flexion contractures

A patient with trigeminal neuralgia has moderate to sever burning and shooting pain. In helping the patient to manage the pain, the nurse recognizes what about this type of pain? a: treatment includes the use of adjuvant analgesics b: will be chronic in nature and require long term treatment c: responds well to small to moderate around-the-clock doses of oral opioids d: can be well controlled with salicylates or NSAIDS

a: treatment includes the use of adjuvant analgesics

A nurse caring for a patient in the ICU implements strategies to create an environment conducive to sleep. Which strategy would be most effective? a: turning off the lights in the room during the night b: having the television on at all times for background noise c: silencing the alarms on the bedside monitor and infusion pumps d: administering ordered analgesics around the clock, even if the patient denies pain.

a: turning off the lights in the room during the night

What should be included in the management during the first 48 hours after an acute soft tissue injury of the ankle (select all that apply) a: use of elastic wrap b: initial immobilization and rest c: elevation of ankle above the heart d: alternating the use of heat and cold e: administration of antiinflammatory drugs

a: use of elastic wrap b: initial immobilization and rest c: elevation of ankle above the heart e: administration of antiinflammatory drugs

2. A patient has been diagnosed with osteosarcoma of the humerus. He shows an understanding of his treatment options when he states a. "I accept that I have to lose my arm with surgery." b. "The chemotherapy before surgery will shrink the tumor." c. "This tumor is related to the melanoma I had 3 years ago." d. "I'm glad they can take out the cancer with such a small scar."

b. "The chemotherapy before surgery will shrink the tumor."

5. A patient is receiving peripheral parenteral nutrition. The parenteral nutrition solution is completed before the new solution arrives on the unit. The nurse administers a. 20% intralipids. b. 5% dextrose solution. c. 0.45% normal saline solution. d. 5% lactated Ringer's solution.

b. 5% dextrose solution.

3. A patient in the unit has a 103.7°F temperature. Which intervention would be most effective in restoring normal body temperature? a. Use a cooling blanket while the patient is febrile. b. Administer antipyretics on an around-the-clock schedule. c. Provide increased fluids and have the UAP give sponge baths. d. Give prescribed antibiotics and provide warm blankets for comfort.

b. Administer antipyretics on an around-the-clock schedule.

What is the rationale for using preoperative checklists on the day of surgery? a. The patient will be correctly identified. b. All preoperative orders and procedures have been carried out and records are complete. c. Patients' families have been informed as to where they can accompany and wait for patients. d. Preoperative medications are the last procedure before the patient is transported to the operating room.

b. All preoperative orders and procedures have been carried out and records are complete.

A patient with a pH of 7.29 has metabolic acidosis. Which value is useful in determining whether the cause of the acidosis is an acid gain or bicarbonate loss? a. PaCO2 b. Anion gap c. Serum Na+ level d. Bicarbonate level

b. Anion gap Calculation of the anion gap by subtracting the serum chloride and bicarbonate levels from the serum sodium level should normally be 10-14 mmol/L. The anion gap is increased in metabolic acidosis associated with acid gain (e.g., diabetic acidosis) but remains normal in metabolic acidosis caused by bicarbonate loss (e.g., diarrhea).

2. The patient tells the nurse in the preoperative setting that she has noticed she has a reaction when wearing rubber gloves. What is the most appropriate intervention? a. Notify the surgeon so the case can be cancelled. b. Ask additional questions to assess for a possible latex allergy. c. Notify the OR staff immediately so that latex-free supplies can be used. d. No intervention is needed because the patient's rubber sensitivity has no bearing on surgery.

b. Ask additional questions to assess for a possible latex allergy.

(CH 41) Which female patient is most likely to have metabolic syndrome a BP 128/78 mm Hg, triglycerides 160 mg/dL, fasting blood glucose 102 mg/dL b. BP 142/90 mm Hg, high-density lipoproteins 45 mg/dL, fasting blood glucose 130 mg/dL c. waist circumference 36 in, triglycerides 162 mg/dL, high density lipoproteins 55 mg/dL d. waist circumference 32 in, high density lipoproteins 38 mg/dL, fasting blood glucose 122 mg/dL

b. BP 142/90 mm Hg, high-density lipoproteins 45 mg/dL, fasting blood glucose 130 mg/dL

3. When administering medications to the patient with gout, the nurse would recognize that which drug is used as a treatment for this disease? a. Colchicine b. Febuxostat c. Sulfasalazine d. Cyclosporine

b. Febuxostat

Which electrolyte imbalance is associated with the following cause? (may be one or more) Primary polydipsia ________ a. Hypernatremia b. Hyponatremia c. Hyperkalemia d. Hypokalemia e. Hypercalcemia f. Hypocalcemia g. Hyperphosphatemia h. Hypophosphatemia i. Hypermagnesemia j. Hypomagnesemia

b. Hyponatremia

While caring for an 84-year-old patient, the nurse monitors the patient's fluid and electrolyte balance, recognizing what as a normal change of aging? a. Hyperkalemia. b. Hyponatremia c. Decreased insensible fluid loss. d. Increased plasma oncotic pressures.

b. Hyponatremia

Which electrolyte imbalance is associated with the following cause? (may be one or more) Loop and thiazide diuretics _____________ a. Hypernatremia b. Hyponatremia c. Hyperkalemia d. Hypokalemia e. Hypercalcemia f. Hypocalcemia g. Hyperphosphatemia h. Hypophosphatemia i. Hypermagnesemia j. Hypomagnesemia

b. Hyponatremia d. Hypokalemia f. Hypocalcemia j. Hypomagnesemia

8. A 65-year-old stroke patient with limited mobility has a purple area of suspected deep tissue injury on the left greater trochanter. Which nursing diagnoses is/are most appropriate (select all that apply)? a. Acute pain related to tissue damage and inflammation b. Impaired skin integrity related to immobility and decreased sensation c. Impaired tissue integrity related to inadequate circulation secondary to pressure d. Risk for infection related to loss of tissue integrity and undernutrition secondary to stroke

b. Impaired skin integrity related to immobility and decreased sensation

5. A nurse is caring for a patient with diabetes who is scheduled for amputation of his necrotic left great toe. The patient's WBC count is 15.0 × 106 /µL, and he has coolness of the lower extremities, weighs 75 lb more than his ideal body weight, and smokes two packs of cigarettes per day. Which priority nursing diagnosis addresses the primary factor affecting the patient's ability to heal? a. Imbalanced nutrition: more than body requirements related to high-fat foods b. Impaired tissue integrity related to decreased blood flow secondary to diabetes and smoking c. Ineffective peripheral tissue perfusion related to narrowed blood vessels secondary to diabetes and smoking d. Ineffective individual coping related to indifference and denial of the long-term effects of diabetes and smoking

b. Impaired tissue integrity related to decreased blood flow secondary to diabetes and smoking

What is hyperkalemia frequently associated with? a. Hypoglycemia b. Metabolic acidosis c. Respiratory alkalosis d. Decreased urine potassium levels

b. Metabolic acidosis

(CH 41) A 40 year old severely obese female patient with type 2 diabetes wants to lose weight. after learning about the surgical procedures, she thinks a combination of restrictive and malabsorptive surgery would be best. which procedure should the nurse teach her about. a. lipectomy b. Roux-en-Y gastric bypass c. adjustable gastric banding d. vertical sleeve gastrectomy

b. Roux-en-Y gastric bypass

4. A nurse is caring for a patient who has a pressure ulcer that is treated with debridement, irrigations, and moist gauze dressings. How should the nurse anticipate healing to occur? a. Tertiary intention b. Secondary intention c. Regeneration of cells d. Remodeling of tissues

b. Secondary intention

The nurse is reviewing a patient's morning laboratory results. Which result is of greatest concern? a. Serum Na+ of 150 mEq/L b. Serum Mg2+ of 1.1 mEq/L c. Serum PO3- of 4.5 mg/dL d. Serum Ca2+ (total) of 8.6 mg/dL

b. Serum Mg2+ of 1.1 mEq/L

7. You are teaching a patient with osteopenia. What is important to include in the teaching plan? a. Lose weight. b. Stop smoking. c. Eat a high-protein diet. d. Start swimming for exercise.

b. Stop smoking.

1. A patient 1 day postoperative after abdominal surgery has incisional pain, 99.5°F temperature, slight erythema at the incision margins, and 30 mL serosanguineous drainage in the Jackson-Pratt drain. Based on this assessment, what conclusion would the nurse make? a. The abdominal incision shows signs of an infection. b. The patient is having a normal inflammatory response. c. The abdominal incision shows signs of impending dehiscence. d. The patient's physician needs to be notified about her condition.

b. The patient is having a normal inflammatory response.

7. A patient on the surgical unit has a history of parasomnia (sleepwalking). What statement describes parasomnia? a. Hypnotic medications reduce the risk of sleepwalking. b. The patient is often unaware of the activity on awakening. c. The patient should be restrained at night to prevent personal harm. d. The potential for sleepwalking is reduced by exercise before sleep.

b. The patient is often unaware of the activity on awakening.

5. In caring for a patient after a spinal fusion, the nurse would immediately report to the physician which patient symptom? a. The patient experiences a single episode of emesis. b. The patient is unable to move the lower extremities. c. The patient is nauseated and has not voided in 4 hours. d. The patient complains of pain at the bone graft donor site.

b. The patient is unable to move the lower extremities.

2. The nurse assessing a patient with a chronic leg wound finds local signs of erythema and pain at the wound site. What would the nurse anticipate being ordered to assess the patient's systemic response? a. Serum protein analysis b. WBC count and differential c. Punch biopsy of center of wound d. Culture and sensitivity of the wound

b. WBC count and differential

5. While caring for a patient with a history of narcolepsy with cataplexy, the nurse can delegate which activity to the unlicensed assistive personnel (UAP)? a. Teaching about the timing of medications b. Walking the patient to and from the bathroom c. Developing a plan of care with a family member d. Planning an appropriate diet that avoids caffeine-containing foods

b. Walking the patient to and from the bathroom

(CH 41) Which patient is at highest risk for complication of obesity? a. a 30-year-old woman who is 5 ft (151 cm) tall, weighs 140lbs (63.6kg), and carries weight in her thighs b. a 56-year-old woman with BMI of 38 kg/m^2, a waist measurement of 38 in (96cm) and a hip measurement of 36 in (91cm) c. a 41-year-old man with a waist measurement of 36in (91cm) and hip measurement of 36 in (91cm) who is 5 ft, 6 in (166cm) tall and weights 150 lb (68.2. kg) d. a 68-year-old man with a waist measurement of 38in (96 cm) and hip measurement of 42in (76cm) who is 5ft, 11in (179cm) tall and weighs 200lb (90.9kg)

b. a 56-year-old woman with BMI of 38 kg/m^2, a waist measurement of 38 in (96cm) and a hip measurement of 36 in (91cm)

9. Diagnostic testing is recommended for skin lesions when a. a health history cannot be obtained. b. a more definitive diagnosis is needed. c. percussion reveals an abnormal finding. d. treatment with prescribed medication has failed.

b. a more definitive diagnosis is needed.

1. Pain is best described as a. a creation of a person's imagination. b. an unpleasant, subjective experience. c. a maladaptive response to a stimulus. d. a neurologic event resulting from activation of nociceptors

b. an unpleasant, subjective experience.

(CH 41) Which statement about obesity is explained by genetics? a. older obese patients have exacerbated changes of aging b. android body shape and weight gain are influence by genetics c. white Americans have a higher incidence of obesity than African Americans d. Men have a harder time losing weight, as they have more muscle mass than women

b. android body shape and weight gain are influence by genetics

6. To detect early signs or symptoms of inadequate oxygenation, the nurse would examine the patient for a. dyspnea and hypotension. b. apprehension and restlessness. c. cyanosis and cool, clammy skin. d. increased urine output and diaphoresis.

b. apprehension and restlessness.

4. The nurse uses knowledge of the effects of stress on the immune system by encouraging patients to a. sleep for 10 to 12 hours per day. b. avoid exposure to upper respiratory tract infections. c. receive regular immunizations when they are stressed. d. use emotion-focused rather than problem-focused coping strategies.

b. avoid exposure to upper respiratory tract infections.

Examples of primary prevention strategies include: a. colonoscopy at age 50. b. avoidance of tobacco products. c. intake of a diet low in saturated fat in a patient with high cholesterol. d. teaching the importance of exercise to a patient with hypertension.

b. avoidance of tobacco products.

9. When auscultating the chest of an older patient in respiratory distress, it is best to a. begin listening at the apices. b. begin listening at the lung bases. c. begin listening on the anterior chest. d. ask the patient to breathe through the nose with the mouth closed.

b. begin listening at the lung bases.

2. A patient has an elevated blood level of indirect (unconjugated) bilirubin. One cause of this finding is that a. the gallbladder is unable to contract to release stored bile. b. bilirubin is not being conjugated and excreted into the bile by the liver. c. the Kupffer cells in the liver are unable to remove bilirubin from the blood. d. there is an obstruction in the biliary tract preventing flow of bile into the small intestine.

b. bilirubin is not being conjugated and excreted into the bile by the liver.

8. When using a prosthesis for transesophageal speech, the patient a. places a vibrating device in the mouth. b. blocks the stoma entrance with a finger. c. swallows air using a Valsalva maneuver. d. places a speaking valve next to the stoma.

b. blocks the stoma entrance with a finger.

3f. The nurse anticipates that treatment of the patient with hyperphosphatemia secondary to renal failure will include a. fluid restriction. b. calcium supplements. c. loop diuretic therapy. d. magnesium supplements

b. calcium supplements.

5. The nurse recognizes that a person who is subjected to chronic stress could be at higher risk for a. osteoporosis. b. colds and flu. c. low blood pressure. d. high serum cholesterol.

b. colds and flu.

(CH 23) a home health nurse is visiting an older obese women who has recently had hip surgery. she tells the patient's caregiver that the patient has intertrigo.When the caregiver asks what that is, the nurse should tell the caregiver that it is: a. thickening of the skin. b. dermatitis in the folds of her skin c. loss of color in diffuse areas of her skin d. a firm plaque caused by fluid int he dermis

b. dermatitis in the folds of her skin

(CH 41) When medication are used in the treatment of obesity, what is most important for the nurse to teach the patient? a. over-the-counter diet aids are safer than other agents and can be useful in controlling appetite b. drugs should be used only as adjuncts to a diet and exercise program as treatment for a chronic condition c. all drugs used for weight control are capable of altering central nervous system function and should be used with caution. d. the primary effect of the medication is psychologic, controlling the urge to eat in response to stress or feelings of rejection.

b. drugs should be used only as adjuncts to a diet and exercise program as treatment for a chronic condition

what covers the larynx during swallowing? a. trachea b. epiglottis c. turbinates d. parietal pleura

b. epiglottis

(CH 23) An active athletic person calls the clinic and describes her feet as having linear breaks through the skin. What is the most likely diagnosis of this problem? a. scales b. fissures c. pustule d. comedo

b. fissures

2. During starvation, the order in which the body obtains substrate for energy is a. visceral protein, skeletal protein, fat, glycogen. b. glycogen, skeletal protein, fat stores, visceral protein. c. visceral protein, fat stores, glycogen, skeletal protein. d. fat stores, skeletal protein, visceral protein, glycogen.

b. glycogen, skeletal protein, fat stores, visceral protein.

5. An 80-year-old man states that, although he adds a lot of salt to his food, it still does not have much taste. The nurse's response is based on the knowledge that the older adult a. should not experience changes in taste. b. has a loss of taste buds, especially for sweet and salt. c. has some loss of taste but no difficulty chewing food. d. loses the sense of taste because the ability to smell is decreased.

b. has a loss of taste buds, especially for sweet and salt.

(CH 41) The nurse admitting a patient for bariatric surgery obtains the following information from the patient. which finding should be brought to the surgeon's attention before proceeding with further patient preparation a. History of hypertension b. history of untreated depression c. history of multiple attempts at weight loss d. history of sleep apnea treated with CPAP

b. history of untreated depression

7. Appropriate nonopioid analgesics for mild pain include (select all that apply) a. oxycodone. b. ibuprofen (Advil). c. lorazepam (Ativan). d. acetaminophen (Tylenol). e. codeine with acetaminophen (Tylenol #3

b. ibuprofen (Advil). d. acetaminophen (Tylenol).

1. In assessing the joints of a patient with osteoarthritis, the nurse understands that Heberden's nodes a. are often red, swollen, and tender. b. indicate osteophyte formation at the DIP joints. c. are the result of pannus formation at the PIP joints. d. occur from deterioration of cartilage by proteolytic enzymes.

b. indicate osteophyte formation at the DIP joints.

9. A patient is scheduled for an electromyogram (EMG). The nurse explains that this diagnostic test involves a. incision or puncture of the joint capsule. b. insertion of small needles into certain muscles. c. administration of a radioisotope before the procedure. d. placement of skin electrodes to record muscle activity.

b. insertion of small needles into certain muscles

(CH 23) The nurse observes that redness remains after palpation of a discolored lesion on the patient's leg. This finding is characteristic of: a. varicosities b. intradermal bleeding. c. dilated blood vessels. d. erythematous lesions.

b. intradermal bleeding.

7. During an examination of the abdomen the nurse should a. position the patient in the supine position with the bed flat and knees straight. b. listen in the epigastrium and all four quadrants for 2 minutes for bowel sounds. c. use the following order of techniques: inspection, palpation, percussion, auscultation. d. describe bowel sounds as absent if no sound is heard in the lower right quadrant after 2 minutes.

b. listen in the epigastrium and all four quadrants for 2 minutes for bowel sounds.

2. A patient with rheumatoid arthritis is experiencing articular involvement of the joints. The nurse recognizes that these characteristic changes include (select all that apply) a. bamboo-shaped fingers. b. metatarsal head dislocation in feet. c. noninflammatory pain in large joints. d. asymmetric involvement of small joints. e. morning stiffness lasting 60 minutes or more.

b. metatarsal head dislocation in feet. e. morning stiffness lasting 60 minutes or more.

6. An example of distraction to provide pain relief is a. TENS. b. music. c. exercise. d. biofeedback

b. music.

(CH 41) a woman is 5 ft, 6 in (166cm) tall and weighs 200lb (90.9kg) with a waist-to-hip ratio of 0.7. the nurse counsels the patient with the knowledge that the patient is at greatest risk for a. heart disease b. osteoporosis c. diabetes mellitus d. endometrial cancer

b. osteoporosis

1. Determination of whether an event is a stressor is based on a person's a. tolerance. b. perception. c. adaptation. d. stubbornness.

b. perception.

2. Activities that the nurse might perform in the role of a scrub nurse during surgery include (select all that apply) a. checking electrical equipment. b. preparing the instrument table. c. passing instruments to the surgeon and assistants. d. coordinating activities occurring in the operating room. e. maintaining accurate counts of sponges, needles, and instruments.

b. preparing the instrument table. c. passing instruments to the surgeon and assistants. e. maintaining accurate counts of sponges, needles, and instruments.

1. The primary function of the skin is a. insulation. b. protection. c. sensation. d. absorption.

b. protection.

3. As gastric contents move into the small intestine, the bowel is normally protected from the acidity of gastric contents by the a. inhibition of secretin release. b. release of bicarbonate by the pancreas. c. release of pancreatic digestive enzymes. d. release of gastrin by the duodenal mucosa.

b. release of bicarbonate by the pancreas.

8. A patient with osteoarthritis is scheduled for a total hip arthroplasty. The nurse explains that the purpose of this procedure is to (select all that apply) a. fuse the joint. b. replace the joint. c. prevent further damage. d. improve or maintain ROM. e. decrease the amount of destruction in the joint.

b. replace the joint. d. improve or maintain ROM. Arthroplasty- the surgical reconstruction or replacement of a joint.

2. Age-related changes in the hair and nails include (select all that apply) a. oily scalp. b. scaly scalp. c. thinner nails. d. thicker, brittle nails. e. longitudinal nail ridging.

b. scaly scalp. d. thicker, brittle nails. e. longitudinal nail ridging.

9. In preparing a patient for a colonoscopy, the nurse explains that a. a signed permit is not necessary. b. sedation may be used during the procedure. c. only one cleansing enema is necessary for preparation. d. a light meal should be eaten the day before the procedure.

b. sedation may be used during the procedure.

(CH 23) A patient has plaque lesion on the dorsal forearm. which type of biopsy is most likely to be used for diagnosis of the lesion? a. punch biopsy b. shave biopsy c. incisional biopsy d. excisional biopsy

b. shave biopsy

(CH 23) The patient asks the nurse what telangiectasia looks like. Which is the best description for the nurse to give the patient? a. a circumscribed, flat discoloration b. small, superficial, dilated blood vessels c. benign tumor of flood or lymph vessels d. tiny purple spots resulting form tiny hemorrhages

b. small, superficial, dilated blood vessels

(CH 41) What characteristics describe adjustable gastric banding (select all that apply) a. 85% of the stomach is removed b. stomach restriction can be reversed c. eliminated hormones that stimulate hunger d. malabsorption of fat-soluble vitamins occur e. inflatable band allows for modification of gastric stoma size f. stomach with gastric pouch surgically anastomosed to the jejunum

b. stomach restriction can be reversed e. inflatable band allows for modification of gastric stoma size

1. Proper attire for the semirestricted area of the surgery department is a. street clothing. b. surgical attire and head cover. c. surgical attire, head cover, and mask. d. street clothing with the addition of shoe covers.

b. surgical attire and head cover.

When does the nurse record the presence of an increased anterioposterior (AP) diameter of the chest? a. there is a prominent protrusion of the sternum b. the width of the chest is equal to the depth of the chest c. there is equal but diminished movement of the two sides of the chest d. the p atient cannot fully expand the lungs because of kyphosis of the spine

b. the width of the chest is equal to the depth of the chest

The patient is complaining of insomnia. Which bedtime snack would be the best option for this patient? a: Hershey's bar b: 8 oz Hot Cocoa c: 8 oz Dannon coffee yogurt d: 1 C. Ben & Jerry's nonfat coffee fudge frozen yogurt

b: 8 oz Hot Cocoa

The patient is told by the healthcare professional that the size of the patient's muscle has decreased. How should the nurse document this occurrence? a: Hyaline b: Atrophy c: Isometric d: Hypertrophy

b: Atrophy

The patient asks, "What does the doctor mean when he says that I have an avulsion fracture in my leg? I thought I had a sprain!" What is the best response by the nurse? a: It is a fracture with more than two fragments b: It means that a ligament pulled a bone fragment loose c: the line of the fracture is twisted along the shaft of the bone d: the line of the fracture is at right angles to the longitudinal axis of the bone

b: It means that a ligament pulled a bone fragment loose

The nurse knows that a patient taught sleep hygiene practices needs further instruction when he says: a: once I go to bed, I should get up if I'm not asleep after 20 minutes b: It's okay to have my usual two glasses of wine in the evening before bed c: a couple of crackers with cheese and some milk may hep to relax before bed d: I should go to they gym earlier in the day so that I'm done at least 6 hours before bedtime.

b: It's okay to have my usual two glasses of wine in the evening before bed

To assess for neurologic status in a patient with with a fractured humerus, what should the nurse ask the patient to do? a: evert, invert, dorsiflex and plantar flex the foot b: abduct, adduct, and oppose the fingers and pronate and supinate the hand c: assess the location, quality, and intensity of pain below the site of the injury assess the color, temperature, cap refill, peripheral pulses and the presence of edema in the extremity.

b: abduct, adduct, and oppose the fingers and pronate and supinate the hand

When positioning a patient after a total hip arthoplasty with a posterior approach, it is important that the nurse maintain the affected extremity in what position? a: adduction and flexion b: abduction and extension c: abduction and internal rotation d: adduction and external rotation

b: abduction and extension

What is the first line therapy for insomnia? a: complementary therapies such as melatonin b: cognitive-behavioral therapies such as relaxation therapy c: benzodiazepine-receptor-like agents (e.g. zolpidem) d: OTC medication such as benadryl

b: cognitive-behavioral therapies such as relaxation therapy

Which measures or drugs may be effective in controlling pain in the physiological pain process stage of transduction? select all that apply a: distraction b: corticosteroids c: epidural opioids d: local anesthetics e: antiseizure medications f: NSAIDS

b: corticosteroids d: local anesthetics e: antiseizure medications f: NSAIDS

What are clinical manifestations of insomnia (select all that apply)? a: narcolepsy b: fragmented sleep c: long sleep latency d: morning headache e: daytime sleepiness f: difficulty concentrating

b: fragmented sleep c: long sleep latency e: daytime sleepiness f: difficulty concentrating

A patient complains of pain in the foot of a leg that was recently amputated. What should the nurse recognize about this pain? a: it is caused by swelling at the incision b: it should be treated with ordered analgesics c: it will become worse with the use of a prosthesis d: it can be managed with diversion because it is psychologic

b: it should be treated with ordered analgesics

In a patient with a stable vertebral fracture, what should the nurse teach the patient to do? a: remain on bed rest until the pain is gone b: logroll to keep the spine straight when turning c: how to use bone cement to correct the problem d: take as much analgesic as needed to relieve the pain

b: logroll to keep the spine straight when turning

The 58 Y.O. male patient will be transferred from the acute care clinical unit of the hospital to another care area. The patient requires complicated dressing changes for several weeks. To which practice setting should the patient be transitioned? a: acute rehabilitation b: long-term acute care c: intermediate care facility d: transitional sub-acute care.

b: long-term acute care

On observation of the patient, the nurse notes the presence of a gait disturbance. How should the nurse further evaluate the patient? a: palpate the hips for crepitation b: measure the length of the limbs c: evaluate the degree of leg movement d: compare the muscle mass of one leg with the other

b: measure the length of the limbs

A patient is discharged from the outpatient clinic following application of a synthetic fiberglass long arm cast for a fractured ulna. Before discharge, the nurse instructs the patient to do what? a: never get the cast wet b: move the shoulder and fingers frequently c: place tape petals around the edges of the cast when it is dry. d: use a sling to support the arm at waist level for the first 48 hours

b: move the shoulder and fingers frequently

A patient is brought to the emergency department with an injured lower left leg following a fall while rock climbing. The nurse identifies the presence of a fracture based on what cardinal sign of fracture? a: muscle spasms b: obvious deformity c: edema and swelling d: pain and tenderness

b: obvious deformity

A patient with severe ulnar deviation of the hands undergoes an arthoplasty with reconstruction and replacement of finger joints. Postoperatively, what is it most important for the nurse to do? a: position the fingers lower than the elbow b: perform neurovascular assessments of the fingers q2-q4 hours c: encourage the patient to gently flex, extend, abduct, and adduct the fingers q4hr d: remind the patient that function of the hands is more important that their cosmetic appearance.

b: perform neurovascular assessments of the fingers q2-q4 hours

A 78 Y.O. female patient is admitted with nausea, vomiting, anorexia, diarrhea and dehydration. She has a history of Diabetes Mellitis and 2 years ago had a stroke with residual right-sided weakness. Identify which characteristics of chronic illness the nurse will probably find in this patient: a: self-limiting b: residual disability c: permanent impairments d: infrequent complications e: need for long-term management f: nonreversible pathological conditions

b: residual disability c: permanent impairments e: need for long-term management f: nonreversible pathological conditions

The patient was referred to the office by the school nurse for a lateral curvature of the spine. The nurse knows this is called: a: lordosis b: scoliosis c: ankylosis d: kyphosis

b: scoliosis

The application of RICE (rest, ice, compression, elevation) is indicated for initial management of which type of injury? a: muscle spasms b: sprains and strains c: repetitive strain injury d: dislocations and subluxations

b: sprains and strains

While caring for an unconscious patient, the nurse discovers a stage II pressure ulcer on the patient's heel. During care of the ulcer, what is the nurse's understanding of a patient's perception of pain? a: the patient will have a behavioral response if pain is perceived b: the area should be treated as a painful lesion, using gentle cleansing and dressing c: the area can be thoroughly scrubbed because the patient is not able to perceive pain d: all nociceptive stimuli that are transmitted to the brain result in the perception of pain.

b: the area should be treated as a painful lesion, using gentle cleansing and dressing

A patient is admitted with an open fracture of the tibia following a bicycle accident. During assessment of the patient, what specifically should the nurse question the patient about? a: any previous injuries to the leg b: the status of the tetanus immunization c: the use of antibiotics in the last month d: whether the injury was exposed to dirt or gravel

b: the status of the tetanus immunization

Following a knee arthoplasty, a patient has a continuous passive motion machine for the affected joint. The nurse explains to the patient that this device is used for what purpose? a: to relieve edema and pain at the incision site b: to promote early joint mobility and increase knee flexion c: to prevent venous stasis and the formation of a DVT d: to improve arterial circulation to the affected extremity to promote healing.

b: to promote early joint mobility and increase knee flexion

A patient with multiple injuries resulting from an automobile accident tells the nurse that he has "bad" pain but that he can "tough it out" and does not require pain medication. To gain the patient's participation in pain management, what should the nurse explain to the patient? a: patients have a responsibility to keep the nurse informed about their pain b: unrelieved pain has many harmful effects on the body that can impair recovery c: using pain medication rarely leads to addiction when they are used for actual pain d: nonpharmacologic therapies can be used to relieve his pain if he is afraid to use pain medications

b: unrelieved pain has many harmful effects on the body that can impair recovery

6. When the nurse is assessing the health perception-health maintenance pattern as related toGI function, an appropriate question to ask is: a. "What is your usual bowel elimination pattern?" b. "What percentage of your income is spent on food?" c. "Have you traveled to a foreign country in the last year?" d. "Do you have diarrhea when you are under a lot of stress?"

c. "Have you traveled to a foreign country in the last year?"

3d. Which patient would be at greatest risk for the potential development of hypermagnesemia? a. 83-year-old man with lung cancer and hypertension b. 65-year-old woman with hypertension taking β-adrenergic blockers c. 42-year-old woman with systemic lupus erythematosus and renal failure d. 50-year-old man with benign prostatic hyperplasia and a urinary tract infection

c. 42-year-old woman with systemic lupus erythematosus and renal failure

What is the normal pH range of the blood and what ratio of base to acid does this reflect? a. 7.32 to 7.42; 25 to 2 b. 7.32 to 7.42; 28 to 2 c. 7.35 to 7.45; 20 to 1 d. 7.35 to 7.45; 30 to 1

c. 7.35 to 7.45; 20 to 1

When the nurse asks a preoperative patient about allergies, the patient reports a history of seasonal environmental allergies and allergies to a variety of fruits. What should the nurse do next? a. Note this information in the patient's record as hay fever and food allergies. b. Place an allergy alert wristband that identifies the specific allergies of the patient. c. Ask the patient to describe the nature and severity of any allergic responses experienced from these agents. d. Notify the anesthesia care provider (ACP) because the patient may have an increased risk for allergies to anesthetics.

c. Ask the patient to describe the nature and severity of any allergic responses experienced from these agents.

1. When a patient is admitted to the PACU, what are the priority interventions the nurse performs? a. Assess the surgical site, noting presence and character of drainage. b. Assess the amount of urine output and the presence of bladder distention. c. Assess for airway patency and quality of respirations, and obtain vital signs. d. Review results of intraoperative laboratory values and medications received.

c. Assess for airway patency and quality of respirations, and obtain vital signs.

5. Which nursing action would be of highest priority when suctioning a patient with a tracheostomy? a. Auscultating lung sounds after suctioning is complete b. Providing a means of communication for the patient during the procedure c. Assessing the patient's oxygenation saturation before, during, and after suctioning d. Administering pain and/or antianxiety medication 30 minutes before suctioning

c. Assessing the patient's oxygenation saturation before, during, and after suctioning

3. Which words are most likely to be used to describe neuropathic pain (select all that apply)? a. Dull b. Mild c. Burning d. Shooting e. Shock-like

c. Burning d. Shooting e. Shock-like

Why does a patient's respoiratory rate increse when there is an excess of carbon dioxide in the blood? a. CO2 displaces oxygen on hemoglobin, leading to a decreased PaO2 b. Co2 causes an increase in the amount of hydrogen ions available in the body c. CO2 combines with water to form carbonic acid, which lowers the pH of cerebrospinal fluid d. Co2 directly stimulates chemoreceptors in the medulla to increase respiratory rate and volume

c. CO2 combines with water to form carbonic acid, which lowers the pH of cerebrospinal fluid

6. Which one of the orders should a nurse question in the plan of care for a patient with a stage III pressure ulcer? a. Pack the ulcer with foam dressing. b. Turn and position the patient every 2 hours. c. Clean the ulcer every shift with Dakin's solution. d. Assess for pain and medicate before dressing change

c. Clean the ulcer every shift with Dakin's solution.

6. In teaching a patient with Sjögren's syndrome about drug therapy for this disorder, the nurse includes instruction on use of which drug? a. Pregabalin (Lyrica) b. Etanercept (Enbrel) c. Cyclosporine (Restasis) d. Cyclobenzaprine (Flexeril)

c. Cyclosporine (Restasis) Sjögren's syndrome: With this disorder, the body's immune system attacks its own healthy cells that produce saliva and tears. Sjögren's often occurs with other such disorders, such as rheumatoid arthritis and lupus. The main symptoms are dry mouth and dry eyes.

A common collaborative problem related to both hyperkalemia and hypokalemia is which potential complication? a. seizures b. Paralysis c. Dysrhythmias d. Acute kidney injury

c. Dysrhythmias

6. During a stressful circumstance that is uncontrollable, which type of coping strategy is the most effective? a. Avoidance b. Coping flexibility c. Emotion-focused coping d. Problem-focused coping

c. Emotion-focused coping

Which electrolyte imbalance is associated with the following cause? (may be one or more) Early burn stage ____________ a. Hypernatremia b. Hyponatremia c. Hyperkalemia d. Hypokalemia e. Hypercalcemia f. Hypocalcemia g. Hyperphosphatemia h. Hypophosphatemia i. Hypermagnesemia j. Hypomagnesemia

c. Hyperkalemia

Which electrolyte imbalance is associated with the following cause? (may be one or more) Chronic kidney disease ___________ a. Hypernatremia b. Hyponatremia c. Hyperkalemia d. Hypokalemia e. Hypercalcemia f. Hypocalcemia g. Hyperphosphatemia h. Hypophosphatemia i. Hypermagnesemia j. Hypomagnesemia

c. Hyperkalemia f. Hypocalcemia h. Hypophosphatemia i. Hypermagnesemia

A patient with chronic kidney disease has hyperphosphatemia. What is the commonly associated electrolyte imbalance? a. Hypokalemia b. Hyponatremia c. Hypocalcemia d. Hypomagnesemia

c. Hypocalcemia

3. The nurse would expect which findings in a patient as a result of the physiologic effect of stress on the reticular formation? a. An episode of diarrhea while awaiting painful dressing changes b. Refusal to communicate with nurses while awaiting a cardiac catheterization c. Inability to sleep the night before beginning to self-administer insulin injections d. Increased blood pressure, decreased urine output, and hyperglycemia after a car accident

c. Inability to sleep the night before beginning to self-administer insulin injections

What is an example of an IV solution that would be appropriate to treat an extracellular fluid volume deficit? a. D5W b. 3% saline c. Lactated Ringer's solution d. D5W in 1/2 normal saline (0.45%)

c. Lactated Ringer's solution

A patient who has a large amount of carbon dioxide in the blood also has what in the blood? a. Large amount of carbonic acid and low hydrogen ion concentration b. small amount of carbonic acid and low hydrogen ion concentration c. Large amount of carbonic acid and high hydrogen ion concentration d. Small amount of carbonic acid and high hydrogen ion concentration

c. Large amount of carbonic acid and high hydrogen ion concentration

1. Which statement best describes the etiology of obesity? a. Obesity primarily results from a genetic predisposition. b. Psychosocial factors can override the effects of genetics in the etiology of obesity. c. Obesity is the result of complex interactions between genetic and environmental factors. d. Genetic factors are more important than environmental factors in the etiology of obesity.

c. Obesity is the result of complex interactions between genetic and environmental factors.

As fluid circulates through the capillaries, there is movement of fluid between the capillaries and the interstitium. What describes the fluid movement that would cause edema? (select all that apply) a. Plasma hydrostatic pressure is less than plasma oncotic pressure. b. Plasma oncotic pressure is higher than interstitial oncotic pressure. c. Plasma hydrostatic pressure is higher than plasma oncotic pressure. d. Plasma hydrostatic pressure is less than interstitial hydrostatic pressure. e. Interstitial hydrostatic pressure is lower than plasma hydrostatic pressure.

c. Plasma hydrostatic pressure is higher than plasma oncotic pressure. e. Interstitial hydrostatic pressure is lower than plasma hydrostatic pressure.

9. An 82-year-old man is being cared for at home by his family. A pressure ulcer on his right buttock measures 1 × 2 × 0.8 cm in depth, and pink subcutaneous tissue is completely visible on the wound bed. Which stage would the nurse document on the wound assessment form? a. Stage I b. Stage II c. Stage III d. Stage IV

c. Stage III

3c. The nurse should be alert for which manifestations in a patient receiving a loop diuretic? a. Restlessness and agitation b. Paresthesias and irritability c. Weak, irregular pulse and poor muscle tone d. Increased blood pressure and muscle spasms

c. Weak, irregular pulse and poor muscle tone

3. A patient is seen at the clinic with fever, muscle aches, sore throat with yellowish exudate, and headache. The nurse anticipates that the collaborative management will include (select all that apply) a. antiviral agents to treat influenza. b. treatment with antibiotics starting ASAP. c. a throat culture or rapid strep antigen test. d. supportive care, including cool, bland liquids. e. comprehensive history to determine possible etiology.

c. a throat culture or rapid strep antigen test. d. supportive care, including cool, bland liquids. e. comprehensive history to determine possible etiology.

3. The nurse can best determine adequate arterial oxygenation of the blood by assessing a. heart rate. b. hemoglobin level. c. arterial oxygen tension. d. arterial carbon dioxide tension.

c. arterial oxygen tension.

8. An important nursing responsibility related to pain is to a. leave the patient alone to rest. b. help the patient appear to not be in pain. c. believe what the patient says about the pain. d. assume responsibility for eliminating the patient's pain.

c. believe what the patient says about the pain.

5. A cancer patient who reports ongoing, constant moderate pain with short periods of severe pain during dressing changes is a. probably exaggerating his pain. b. best treated by referral for surgical treatment of his pain. c. best treated by receiving both a long-acting and a short-acting opioid. d. best treated by regularly scheduled short-acting opioids plus acetaminophen.

c. best treated by receiving both a long-acting and a short-acting opioid.

A pulse oximetry monitor indicates the patient has a drop in SpO2 from 95% to 85% over several hours. What is the first action the nurse should take? a. order STAT ABGs to confirm SpO2 with SaO2. b. start oxygen administration by nasal cannula at 2 L/min c. check the position of the probe on the finger or earlobe d. notify the health provider of the change in baseline PaO2.

c. check the position of the probe on the finger or earlobe

2. A patient is receiving a PCA infusion after surgery to repair a hip fracture. She is sleeping soundly but awakens when the nurse speaks to her in a normal tone of voice. Her respirations are 8 breaths/minute. The most appropriate nursing action in this situation is to a. stop the PCA infusion. b. obtain an oxygen saturation level. c. continue to closely monitor the patient. d. administer naloxone and contact the physician

c. continue to closely monitor the patient.

4. Unrelieved pain is a. expected after major surgery. b. expected in a person with cancer. c. dangerous and can lead to many physical and psychologic complications. d. an annoying sensation, but it is not as important as other physical care needs.

c. dangerous and can lead to many physical and psychologic complications.

(CH 23) When the nurse is assessing the skin of an older adult, which factor is likely to contribute to dry skin? a. increased bruising b. excessive perspiration c. decreased extracellular fluid d. decreased peripheral blood supply

c. decreased extracellular fluid

4. An indication of a neurovascular problem noted during assessment of the patient with a fracture is a. exaggeration of strength with movement. b. increased redness and heat below the injury. c. decreased sensation distal to the fracture site. d. purulent drainage at the site of an open fracture.

c. decreased sensation distal to the fracture site.

6. While obtaining subjective assessment data related to the musculoskeletal system, it is particularly important to ask a patient about other medical problems such as a. hypertension. b. thyroid problems. c. diabetes mellitus. d. chronic bronchitis

c. diabetes mellitus.

A common reason that a nurse may need extra time when preparing the older adult for surgery is their a. ineffective coping b. limited adaptation to stress c. diminished vision and hearing d. need to include caregivers in activities

c. diminished vision and hearing

(CH 41) Which explanation about weight reduction should be included when teaching the obese patient and her obese husband? a. weight gain is caused by psychologic factors b. daily weighing is recommended to monitor weight loss c. fat is not burned until the glycogen-water pool is depleted d. med lose weight less quickly that women because they have a higher percentage of metabolically less-active fat

c. fat is not burned until the glycogen-water pool is depleted

Pulse oximetry may not be a reliable indicator of oxygen saturation in a patient with: a. fever b. anesthetized c. hypovolemic shock d. receiving oxygen therapy

c. hypovolemic shock

7. During the respiratory assessment of the older adult, the nurse would expect to find (select all that apply) a. a vigorous cough. b. increased chest expansion. c. increased residual volume. d. increased breath sounds in the lung apices. e. increased anteroposterior (AP) chest diameter.

c. increased residual volume. e. increased anteroposterior (AP) chest diameter.

7. Intravenous induction for general anesthesia is the method of choice for most patients because a. the patient is not intubated. b. the agents are nonexplosive. c. induction is rapid and pleasant. d. emergence is longer but with fewer complications.

c. induction is rapid and pleasant.

(CH 41) What is the main underlying risk factor for metabolic syndrome a. age b. heart disease c. insulin resistance d. high cholesterol levels

c. insulin resistance

9. Providing opioids to a dying patient who is experiencing moderate to severe pain a. may cause addiction. b. will probably be ineffective. c. is an appropriate nursing action. d. will likely hasten the person's death.

c. is an appropriate nursing action.

6. The typical fluid replacement for the patient with a fluid volume deficit is a. dextran. b. 0.45% saline. c. lactated Ringer's. d. 5% dextrose in 0.45% saline.

c. lactated Ringer's.

Ageism is characterized by a. denial of negative stereotypes regarding aging. b. positive attitudes toward the elderly based on age. c. negative attitudes toward the elderly based on age. d. negative attitudes toward the elderly based on physical disability.

c. negative attitudes toward the elderly based on age.

5. Discharge criteria for the Phase II patient include (select all that apply) a. no nausea or vomiting. b. ability to drive self home. c. no respiratory depression. d. written discharge instructions understood. e. opioid pain medication given 45 minutes ago

c. no respiratory depression. d. written discharge instructions understood. e. opioid pain medication given 45 minutes ago

1. The bone cells that function in the resorption of bone tissue are called a. osteoids. b. osteocytes. c. osteoclasts. d. osteoblasts.

c. osteoclasts.

4. During the physical examination of a patient's skin, the nurse would a. use a flashlight in a poorly lit room. b. note cool, moist skin as a normal finding. c. pinch up a fold of skin to assess for turgor. d. perform a lesion-specific examination first and then a general inspection.

c. pinch up a fold of skin to assess for turgor.

How does the nurse assess the patient's chest expansion? a. put the palms of hands against the chest wall b. put the index fingers on either side of the trachea c. place the thumbs at the midline of the lower chest d. place one hand on the lower anterior chest and one hand on the upper abdomen

c. place the thumbs at the midline of the lower chest

(CH 41) During care of the severely obese patient, what is most important for the nurse to do? a. avoid reference to the patient's weight to avoid embarrassing the patient b. emphasize to the patient how important it is to lose weight to maintain health c. plan for necessary modification in equipment and nursing techniques before initiating care d. recognize that a full assessment of each body system might not be possible because of numerous layers of skinfolds

c. plan for necessary modification in equipment and nursing techniques before initiating care

2. A patient is admitted to the PACU after major abdominal surgery. During the initial assessment the patient tells the nurse he thinks he is going to "throw up." A priority nursing intervention would be to a. increase the rate of the IV fluids. b. obtain vital signs, including O2 saturation. c. position patient in lateral recovery position. d. administer antiemetic medication as ordered.

c. position patient in lateral recovery position.

3. When assessing the nutritional-metabolic pattern in relation to the skin, the nurse questions the patient regarding a. joint pain. b. the use of moisturizing shampoo. c. recent changes in wound healing. d. self-care habits related to daily hygiene.

c. recent changes in wound healing.

8. Preoperative considerations for older adults include (select all that apply) a. only using large-print educational materials. b. speaking louder for patients with hearing aids. c. recognizing that sensory deficits may be present. d. providing warm blankets to prevent hypothermia. e. teaching important information early in the morning.

c. recognizing that sensory deficits may be present. d. providing warm blankets to prevent hypothermia. e. teaching important information early in the morning.

A characteristic of a chronic illness is that it (select all that apply) a. has reversible pathologic changes. b. has a consistent, predictable clinical course. c. results in permanent deviation from normal. d. is associated with many stable and unstable phases. e. always starts with an acute illness and then progresses slowly.

c. results in permanent deviation from normal. d. is associated with many stable and unstable phases.

(CH 41) In preparing to care for the obese patient with cancer, what physiologic problems is this patient at a greater risk for having (select all that apply) a. tinnitus b. fractures c. sleep apnea d. trousseau's sign e. type 2 diabetes mellitus f. gastroesophogeal reflux disease (GERD)

c. sleep apnea e. type 2 diabetes mellitus f. gastroesophogeal reflux disease (GERD)

8. Strategies to reduce sleepiness during nighttime working include a. exercising before work. b. taking melatonin before working the night shift. c. sleeping for at least 2 hours immediately before work time. d. walking for 10 minutes every 4 hours during the night shift.

c. sleeping for at least 2 hours immediately before work time.

What is a primary nursing responsibility after obtaining a blood specimen for ABGs? a. adding heparin to the blood specimen b. applying pressure to the puncture site for a full 2 mintues c. taking the specimen immediately to the laboratory in an iced container d. avoiding any changes in oxygen intervention for 20 minutes following the procedure

c. taking the specimen immediately to the laboratory in an iced container

Nursing interventions directed at health promotion in the older adult are primarily focused on a. disease management. b. controlling symptoms of illness. c. teaching positive health behaviors. d. teaching regarding nutrition to enhance longevity.

c. teaching positive health behaviors.

What accurately describes the alveolar sacs? a. line the lung pleura b. warm and moisturize inhaled air c. terminal structures of the respiratory tract d. contain dead air that is not available for gas exchange

c. terminal structures of the respiratory tract

An important nursing action to help a chronically ill older adult is to a. avoid discussing future lifestyle changes. b. assure the patient that the condition is stable. c. treat the patient as a competent manager of the disease. d. encourage the patient to "fight" the disease as long as possible.

c. treat the patient as a competent manager of the disease.

During muscle strength testing, the patient has active movement against gravity and some resistance to pressure. What score should the nurse give this finding? a: 2 b: 3 c: 4 d: 5

c: 4

When preparing a patient for discharge following fixation of a mandibular fracture , the nurse determines that the teaching has been successful when the patient says: a: I can keep my mouth moist bu sucking on hard candy b: I should cut the wires with scissors if I begin to vomit c: I may use a bulk-forming laxative if my liquid diet causes constipation d: I should use a moist swab to clean my mouth every time I eat something

c: I may use a bulk-forming laxative if my liquid diet causes constipation

Amitriptyline (Elavil) is prescribed for a patient with chronic pain from fibromyalgia. When the nurse explains that this drug is an antidepressant, the patient states that she is in pain, not depressed. What is the nurse's best response to the patient? a: antidepressants will improve the patient's attitude and prevent a negative emotional response to pain b: chronic pain almost always leads to depression, and the use of this drug will prevent depression from occurring. c: Some antidepressant medications relieve pain by releasing neurotransmitters that prevent pain impulses from reaching the brain d: certain antidepressant drugs are metabolized in the liver to substances that numb the ends of the nerve fibers, preventing the onset of pain.

c: Some antidepressant medications relieve pain by releasing neurotransmitters that prevent pain impulses from reaching the brain

Pain is defined as "whatever the person experiencing the pain says it is, existing whenever the patient says it does." This definition is problematic for the nurse when caring for which type of patient? a: a patient placed on a ventilator b: a patient with a history of opioid addiction c: a patient with decreased cognitive function d: a patient with pain resulting from severe trauma

c: a patient with decreased cognitive function

The woman with osteoporosis slipped on the ice and now her wrist hurts. If there is a fracture, what type of fracture is expected? a: dislocation b: open fracture c: colles' fracture d: incomplete fracture

c: colles' fracture

A patient with chronic cancer-related pain has started using MS contin for pain control and has developed common side effects of the drug. The nurse reassures the patient that tolerance will develop to most of these side effects but that continued treatment will most likely be required for what? a: pruritis b: dizziness c: constipation d: nausea and vomiting

c: constipation

On the first postoperative day following a bowel resection, the patient complains of abdominal and incisional pain rated 9 on a scale of 0-10. Postoperative orders include morphine 4mg IV q2hr for pain and may repeat morphine 4mg IV for breakthrough pain. The nurse determines that it has been only 2 hours since the last dose of morphine and wants to wait a little longer. What effect does the nurse's action have on the patient? a: protects the patient from addiction and toxic effects of the drug b: prevents hastening or causing a patient's death from respiratory dysfunction c: contributes to unnecessary suffering and physical and psychosocial dysfunction d: indicates that the nurse understands the adage of "start low and go slow" in administering analgesics.

c: contributes to unnecessary suffering and physical and psychosocial dysfunction

Which joint surgery is used to arthroscopically remove degenerative tissue in joints? a: osteotomy b: arthrodesis c: debridement d: synovectomy

c: debridement

A patient with a fractured tibia accompanied by extensive soft tissue damage initially has a splint applied and held in place with an elastic bandage. What early sign should alert the nurse that the patient is developing compartment syndrome? a: paralysis of the toes b: absence of peripheral pulses c: distal pain unrelieved by opioid analgesics d: skin over the injury site is blanched when the bandage is removed.

c: distal pain unrelieved by opioid analgesics

Which statement about sleep is accurate? a: lack of sleep causes medical and psychiatric disorders b: adults generally require at least 5 hours of sleep every 24 hours c: during sleep an individual is not consciously aware of his or her environment d: less than 10% of adults report at least one sleep problem, such as difficulty falling asleep

c: during sleep an individual is not consciously aware of his or her environment

Which type of fracture occurred when there is radial nerve and brachial artery damage and the fracture is reduced with a hanging arm cast? a: fractured tibia b: colles' fracture c: fractured humerus d: femoral shaft fracture

c: fractured humerus

A patient with severe joint immobility is receiving physical and exercise therapy. To evaluate the effect of the treatment, the nurse may assess joint range of motion with what equipment? a: ergometer b: myometer c: goniometer d: arthrometer

c: goniometer

The athlete comes to the clinic with bursitis. What does the nurse know happens to the tissue to cause pain when bursitis occurs? a: tearing of a ligament b: stretchingof muscle and fascia sheath c: inflammation of synovial membrane sac at friction sites d: incomplete separation of articular surfaces of joint cause by ligament injury

c: inflammation of synovial membrane sac at friction sites

What controls the cyclic changes between waking and sleep? a: fluctuating levels of melatonin b: the environmental light-dark cycles c: key nuclei in the brainstem, hypothalamus and thalamus d: a variety of neuropeptides released from the nervous system

c: key nuclei in the brainstem, hypothalamus and thalamus

An 88 Y.O. woman is brought to the health clinic for the first time by her 64 Y.O. daughter. During the initial comprehensive nursing assessment of the patient, what should the nurse do? a: ask the daughter whether the patient has any urgent needs or problems b: interview the patient and daughter together so that pertinent information can be obtained c: obtain a health history using a functional health pattern and assess activities of daily living (ADLs) and mental status d: refer the patient for an interdisciplinary comprehensive geriatric assessment because at her age she will have multiple needs.

c: obtain a health history using a functional health pattern and assess activities of daily living (ADLs) and mental status

Which type of bone cell is responsible for the formation of bone? a: osteocyte b: osteoclast c: osteoblast d: sarcomere

c: osteoblast

The patient with osteoporosis had a spontaneous hip fracture. How should the nurse document this before the x-ray results return? a: open fracture b: oblique fracture c: pathologic fracture d: greenstick fracture

c: pathologic fracture

The nurse teaches individuals that one of the best ways to prevent musculoskeletal injuries during physical exercise is by doing what? a: increase muscle strength with daily isometric exercise b: avoid exercising on concrete or hard pavement surfaces. c: perform stretching and warm up exercises before exercise d: wrap susceptible joints with elastic bandages or adhesive tape before exercise

c: perform stretching and warm up exercises before exercise

What is an important nursing measure in the rehabilitation of an older adult to prevent loss of function from inactivity and immobility. a: using assistive devices such as walkers and canes b: teaching good nutrition to prevent loss of muscle mass c: performance of active and passive ROM exercises d: performance of risk appraisals and assessments related to immobility

c: performance of active and passive ROM exercises

The nurse suspects a fat embolism rather than a pulmonary embolism from a venous thrombosis when the patient with a fracture develops what? a: tachycardia and dyspnea b: a sudden onset of chest pain c: peteciae around the neck and upper chest d: electrocardiograph (ECG) changes and decreased O2

c: peteciae around the neck and upper chest

The patient has a burning, sharp pain on the sole of the foot, especially in the morning. The nurse knows that this describes what common musculoskeletal problem? a: pes planus b: tenosynovitis c: plantar fascitis d: muscle atrophy

c: plantar fascitis -Pas planus- A condition in which the entire sole of the foot touches the floor when standing. S/S: Most people have no symptoms associated with flat feet. But some people with flat feet experience foot pain, particularly in the heel or arch area. -Tenosynovitis- Inflammation of the tendon sheath where muscle connects to bone. S/S: Symptoms include pain, swelling, and difficulty moving the affected joint. -Plantar fascitis- An inflammation of a thick band of tissue that connects the heel bone to the toes. S/S: Symptoms include stabbing pain near the heel. Pain might be worst in the morning.

The physician has order CPAP (continuous positive airway pressure) for a patient with serious obstructive sleep apnea. How will the CPAP help the patient? a: prevent airway occlusion by bringing the tongue forward b: be easily tolerated by both the patient and the patient's bed partner c: provide enough positive pressure in the airway to prevent airway collapse d: deliver a high inspiratory pressure and a low expiratory pressure to prevent airway collapse.

c: provide enough positive pressure in the airway to prevent airway collapse

Twenty-four hours after a below-the-knee amputation, a patient uses the call system to tell the nurse that his dressing (a compression bandage) has fallen off. What is the first action that the nurse should take? a: apply ice to the site b: cover the incision with dry gauze c: reapply compression dressings d: elevate the extremity on a couple of pillows

c: reapply compression dressings

When the nursing student asks the RN what an arthoplasty is, what is the best description the RN can give the student? a: surgical fusion of the joint to relieve ain b: correction of bone deformity by removal of a wedge or slice of bone c: reconstruction or replacement of a joint to relieve pain and correct deformity d: used in rheumatoid arthritis to remove the tissue involved in joint destruction

c: reconstruction or replacement of a joint to relieve pain and correct deformity

A patient with an extracapsular hip fracture is admitted to the orthopedic unit and placed in Buck's traction. The nurse explains to the patient that the purpose of the traction is to do what? a: pull bone fragments back into alignment b: immobilize the leg until healing is complete c: reduce pain and muscle spasms before surgery d: prevent damage to the blood vessels at the fracture site

c: reduce pain and muscle spasms before surgery Bucks traction or Russel's traction are both types of skin traction.

Which serologic tests would be done to evaluate rheumatoid arthritis (select all that apply) a: uric acid b: anti-DNA antibody c: rheumatoid factor (RF) d: antinuclear antibody (ANA) e: erythrocyte sedimentation rate (ESR)

c: rheumatoid factor (RF) d: antinuclear antibody (ANA) e: erythrocyte sedimentation rate (ESR)

In report, the nurse is told that the patient has a contracture of the right arm. What does the nurse know this means? a: a fluid filled cyst b: generalized muscle pain c: shortening of the muscle or ligament d: grating sensation between bones with movement

c: shortening of the muscle or ligament

An 83 Y.O. woman is being discharged from the hospital following stabilization of her international normalized ratio (INR) levels (used to assess effectiveness of warfarin therapy). She has chronic atrial fibrillation and has been on Warfarin for several years. Discharge instructions include returning to the clinic weekly for INR testing. Which statement by the patient indicates that she may not be able to get the testing done? a: when I have the energy, I have taken the bus to get this test done b: I will need to ask my son to bring me into town every week for the test c: should I just keep taking the same pill every day until I can get a ride into town? d: it is very important to have this test every week. I have several church friends who can bring me

c: should I just keep taking the same pill every day until I can get a ride into town?

What is the most common diagnostic test used to assess musculoskeletal disorders? a: myelogram b: arthroscopy c: standard x-ray d: magnetic resonance imaging (MRI)

c: standard x-ray

(CH 41) The nurse has completed initial instructions with a patient regarding a weight loss program. which patient comment indicates to the nurse that the teaching has been effective a. "i will keep a diary of daily weight to illustrate my weight loss." b. "i plan to lose 4 lbs a week until i have lost 60 lbs i want to lose" c. "I should not exercise more than what is required so i don't increase my appetite" d. "I plan to join a behavior modification group to help establish long term behavior changes"

d. "I plan to join a behavior modification group to help establish long term behavior changes"

5. This bariatric surgical procedure involves creating a stoma and gastric pouch that is reversible, and no malabsorption occurs. What surgical procedure is this? a. Vertical gastric banding b. Biliopancreatic diversion c. Roux-en-Y gastric bypass d. Adjustable gastric banding

d. Adjustable gastric banding

During a preoperative review of systems, the patient reveals a history of renal disease. This finding suggests the need for which preoperative diagnostic tests? a. ECG and chest X-ray b. Serum glucose and CBC c. ABGs and coagulation tests d. BUN, serum creatinine, and electrolytes

d. BUN, serum creatinine, and electrolytes

10. Which assessment finding of the respiratory system does the nurse interpret as abnormal? a. Inspiratory chest expansion of 1 in b. Percussion resonance over the lung bases c. Symmetric chest expansion and contraction d. Bronchial breath sounds in the lower lung fields

d. Bronchial breath sounds in the lower lung fields

In a patient with positive Chvostek's sign, the nurse would anticipate the IV administration of which medications? a. Calcitonin b. Vitamin D c. Loop diuretics d. Calcium gluconate

d. Calcium gluconate -Chovstek's Sign- twitching of the facial muscles in response to tapping over the area of the facial nerve. -Trousseau's sign is carpopedal spasm caused by inflating the blood-pressure cuff to a level above systolic pressure for 3 minutes.

In a patient with sodium imbalances, the primary clinical manifestations are related to alterations in what body system? a. Kidneys b. Cardiovascular system c. Musculoskeletal system d. Central nervous system

d. Central nervous system

7. While in the recovery room, a patient with a total laryngectomy is suctioned and has bloody mucus with some clots. Which nursing interventions would apply? a. Notify the physician immediately. b. Place the patient in the prone position to facilitate drainage. c. Instill 3 mL of normal saline into the tracheostomy tube to loosen secretions. d. Continue your assessment of the patient, including O2 saturation, respiratory rate, and breath sounds.

d. Continue your assessment of the patient, including O2 saturation, respiratory rate, and breath sounds.

What is the compensatory mechanism for metabolic alkalosis? a. Shifting of bicarbonate into cells in exchange for chloride b. Kidney conservation of bicarbonate and excretion of hydrogen ions c. Deep, rapid respirations (Kussmaul respirations) to increase CO2 excretion d. Decreased respiratory rate and depth to retain CO2 and kidney excretion of bicarbonate

d. Decreased respiratory rate and depth to retain CO2 and kidney excretion of bicarbonate

What stimulates aldosterone secretion from the adrenal cortex? a. Excessive water intake. b. Increased serum osmolality. c. Decreased serum potassium. d. Decreased sodium and water.

d. Decreased sodium and water.

Which patient is at risk for hypernatremia? a. Has a deficiency of aldosterone b. Has prolonged diarrhea c. Receives excessive IV 5% dextrose solution d. Has impaired consciousness and decreased thirst sensitivity.

d. Has impaired consciousness and decreased thirst sensitivity.

Which electrolyte imbalance is associated with the following cause? (may be one or more) Metabolic alkalosis ________ a. Hypernatremia b. Hyponatremia c. Hyperkalemia d. Hypokalemia e. Hypercalcemia f. Hypocalcemia g. Hyperphosphatemia h. Hypophosphatemia i. Hypermagnesemia j. Hypomagnesemia

d. Hypokalemia

6. A patient with anorexia nervosa shows signs of malnutrition. During initial refeeding, the nurse carefully assesses the patient for a. hyperkalemia. b. hypoglycemia. c. hypercalcemia. d. Hypophosphatemia.

d. Hypophosphatemia.

A patient is taking diuretic drugs that cause sodium loss from the kidney. Which fluid or electrolyte imbalance is most likely to occur in this patient? a. Hyperkalemia b. Hyponatremia c. Hypocalcemia d. Hypotonic fluid loss

d. Hypotonic fluid loss

Which procedures are done for curative purposes? (select all that apply) a. Gastroscopy b. Rhinoplasty c. Tracheotomy d. Hysterectomy e. Herniorrhaphy

d. Hysterectomy e. Herniorrhaphy

8. On inspection of a patient's dark skin, the nurse notes a blue-gray birthmark on the forehead and eye area. This assessment finding is called a. vitiligo. b. intertrigo. c. telangiectasia. d. Nevus of Ota.

d. Nevus of Ota. - Vitiligo: A disease that causes the loss of skin color in blotches. - Intertrigo: a rash that shows up between skin folds. Very common - Telangiectasia: a condition characterized by dilation of the capillaries, which causes them to appear as small red or purple clusters, often spidery in appearance, on the skin or the surface of an organ.

On assessment of a central venous access device (CVAD) site, the nurse observes that the transparent dressing is loose along two sides. What should the nurse do immediately? a. Wait and change the dressing when it is due. b. Tape the two loose sides down and document. c. Apply a gauze dressing over the transparent dressing and tape securely. d. Remove the dressing and apply a new transparent dressing using sterile technique.

d. Remove the dressing and apply a new transparent dressing using sterile technique.

A nurse has beenexposed to tuberculosis during crae of a patient with TB and has a TB skin test performed. When is the nurse considered infected? a. There is no redness or induration at the injection site b. There is an induration of only 5 mm at the injection site c. A negative skin test is followed by a negative chest x-ray d. Testing causes a 10 mm red, indurated area at the injection site

d. Testing causes a 10 mm red, indurated area at the injection site

2. Which statement is true regarding rapid eye movement (REM) sleep? a. The EEG pattern is quiescent. b. It occurs only once in the night. c. It is separated by distinct physiologic stages. d. The most vivid dreaming occurs during this phase.

d. The most vivid dreaming occurs during this phase.

The nurse is reviewing the laboratory results for a preoperative patient. Which test result should be brought to the attention of the surgeon immediately? a. Serum K+ of 3.8 mEq/L b. Hemoglobin of 15 g/dL c. Blood glucose of 100 mg/dL d. White blood cell (WBC) count of 18,500/microliter

d. White blood cell (WBC) count of 18,500/microliter

10. A nurse believes that patients with the same type of tissue injury should have the same amount of pain. This statement reflects a. a belief that will contribute to appropriate pain management. b. an accurate statement about pain mechanisms and an expected goal of pain therapy. c. a belief that will have no effect on the type of care provided to people in pain. d. a lack of knowledge about pain mechanisms, which is likely to contribute to poor pain management.

d. a lack of knowledge about pain mechanisms, which is likely to contribute to poor pain management.

How is the presence of bronchovesicular sounds in the peripheral lung fields described? a. rhonchi b. crackles c. adventitious sounds d. abnormal lung sounds

d. abnormal lung sounds

7. When grading muscle strength, the nurse records a score of 3, which indicates a. no detection of muscular contraction. b. a barely detectable flicker of contraction. c. active movement against full resistance without fatigue. d. active movement against gravity but not against resistance.

d. active movement against gravity but not against resistance.

3. A patient with a comminuted fracture of the tibia is to have an open reduction with internal fixation (ORIF) of the fracture. The nurse explains that ORIF is indicated when a. the patient is unable to tolerate prolonged immobilization. b. the patient cannot tolerate the surgery of a closed reduction. c. a temporary cast would be too unstable to provide normal mobility. d. adequate alignment cannot be obtained by other nonsurgical methods.

d. adequate alignment cannot be obtained by other nonsurgical methods.

Older adults who become ill are more likely than younger adults to a. complain about the symptoms of their problems. b. refuse to carry out lifestyle changes to promote recovery. c. seek medical attention because of limitations on their lifestyle. d. alter their daily living activities to accommodate new symptoms.

d. alter their daily living activities to accommodate new symptoms.

(CH 23) What is the primary difference between an excoriation and an ulcer? a. ulcers do not penetrate below the epidermal junction b. excoriations involve only thinning of the epidermis and dermis. c. excoriations will form crusts or scabs whereas ulcers remain open d. an excoriation heals without scarring because the dermis is not involved.

d. an excoriation heals without scarring because the dermis is not involved.

2. The obesity classification that is most often associated with cardiovascular health problems is a. primary obesity. b. secondary obesity. c. gynoid fat distribution. d. android fat distribution.

d. android fat distribution.

(CH 23) When performing a physical assessment of the skin, what should the nurse do first? a. palpate the temperature of the skin with the fingertips b. assess the degree of turgor by pinching the skin on the forearm c. inspect specific lesions before performing a general examination of the skin d. ask the patient to undress completely so all areas of the skin can be inspected

d. ask the patient to undress completely so all areas of the skin can be inspected

7. An appropriate nursing intervention for a hospitalized patient who states she cannot cope with her illness is a. controlling the environment to prevent sensory overload and promote sleep. b. encouraging the patient's family to offer emotional support by frequent visiting. c. arranging for the patient to phone family and friends to maintain emotional bonds. d. asking the patient to describe previous stressful situations and how she managed to resolve them.

d. asking the patient to describe previous stressful situations and how she managed to resolve them.

1. A patient was seen in the clinic for an episode of epistaxis, which was controlled by placement of anterior nasal packing. During discharge teaching, the nurse instructs the patient to: a. use aspirin for pain relief. b. remove the packing later that day. c. skip the next dose of antihypertensive medication. d. avoid vigorous nose blowing and strenuous activity.

d. avoid vigorous nose blowing and strenuous activity.

4. When teaching the patient with primary insomnia about sleep hygiene, the nurse should emphasize a. the importance of daytime naps. b. the need to exercise before bedtime. c. the need for long-term use of hypnotics. d. avoiding caffeine-containing beverages 6 to 9 hours before bedtime.

d. avoiding caffeine-containing beverages 6 to 9 hours before bedtime.

9. In teaching a patient scheduled for a total ankle replacement, it is important to tell the patient that after surgery he should avoid a. lifting heavy objects. b. sleeping on the back. c. abduction exercises of the affected ankle. d. bearing weight on the affected leg for 6 weeks.

d. bearing weight on the affected leg for 6 weeks.

(CH 41) when teaching a patient about weight reduction diets, the nurse teaches the patient that an appropriate single serving of food is a. a 6 in bagel b. 1 cup of chopped vegetables c. piece of cheese the size of three dice d. chicken breast the size of a deck of cards

d. chicken breast the size of a deck of cards

A 73-ear old patient has an SpO2 of 70%. What other assessment should the nurse consider before making a judgment about the adequacy of the patient's oxygenation? a. what the oxygenation status is with a stress test b. trend and rate of development of the hyperkalemia c. comparison of patient's spo2 values with normal values d. comparison of patient's current vital signs with normal vital signs

d. comparison of patient's current vital signs with normal vital signs

2. A patient with a respiratory condition asks "How does air get into my lungs?" The nurse bases her answer on her knowledge that air moves into the lungs because of a. contraction of the accessory abdominal muscles. b. increased carbon dioxide and decreased oxygen in the blood. c. stimulation of the respiratory muscles by the chemoreceptors. d. decrease in intrathoracic pressure relative to pressure at the airway.

d. decrease in intrathoracic pressure relative to pressure at the airway.

An ethnic older adult may feel a loss of self-worth when the nurse a. informs the patient about ethnic support services. b. allows a patient to rely on ethnic health beliefs and practices. c. has to use an interpreter to provide explanations and teaching. d. emphasizes that a therapeutic diet does not allow ethnic foods.

d. emphasizes that a therapeutic diet does not allow ethnic foods.

4. A 70-kg postoperative patient has an average urine output of 25 mL/hr during the first 8 hours. The priority nursing intervention(s) given this assessment would be to a. perform a straight catheterization to measure the amount of urine in the bladder. b. notify the physician and anticipate obtaining blood work to evaluate renal function. c. continue to monitor the patient because this is a normal finding during this time period. d. evaluate the patient's fluid volume status since surgery and obtain a bladder ultrasound

d. evaluate the patient's fluid volume status since surgery and obtain a bladder ultrasound

5. A priority nursing intervention to assist a preoperative patient in coping with fear of postoperative pain would be to a. inform the patient that pain medication will be available. b. teach the patient to use guided imagery to help manage pain. c. describe the type of pain expected with the patient's particular surgery. d. explain the pain management plan, including the use of a pain rating scale.

d. explain the pain management plan, including the use of a pain rating scale.

4. The best nutritional therapy plan for a person who is obese is a. the Zone diet. b. the Atkins diet. c. Sugar Busters. d. foods from the basic food groups.

d. foods from the basic food groups.

7. A patient who normally takes 40 units of glargine insulin (long acting) at bedtime asks the nurse what to do about her dose the night before surgery. The best response would be to have her a. skip her insulin altogether the night before surgery. b. take her usual dose at bedtime and eat a light breakfast in the morning. c. eat a moderate meal before bedtime and then take half her usual insulin dose. d. get instructions from her surgeon or health care provider on any insulin adjustments.

d. get instructions from her surgeon or health care provider on any insulin adjustments.

5. When scrubbing at the scrub sink, the nurse should a. scrub from elbows to hands. b. scrub without mechanical friction. c. scrub for a minimum of 10 minutes. d. hold the hands higher than the elbows.

d. hold the hands higher than the elbows.

7. The nurse is unable to flush a central venous access device and suspects occlusion. The best nursing intervention would be to a. apply warm moist compresses to the insertion site. b. attempt to force 10 mL of normal saline into the device. c. place the patient on the left side with head-down position. d. instruct the patient to change positions, raise arm, and cough.

d. instruct the patient to change positions, raise arm, and cough.

3. To prevent muscle atrophy, the nurse teaches the patient with a leg immobilized in traction to perform (select all that apply) a. flexion contractions. b. tetanic contractions. c. isotonic contractions. d. isometric contractions. e. extension contractions.

d. isometric contractions.

2. A patient with allergic rhinitis reports severe nasal congestion; sneezing; and watery, itchy eyes and nose at various times of the year. To teach the patient to control these symptoms, the nurse advises the patient to a. avoid all intranasal sprays and oral antihistamines. b. limit the usage of nasal decongestant spray to 10 days. c. use oral decongestants at bedtime to prevent symptoms during the night. d. keep a diary of when the allergic reaction occurs and what precipitates it.

d. keep a diary of when the allergic reaction occurs and what precipitates it.

Palpation is the assessment technique used to find which abnormal assessment findings? Select all a. stridor b. finger clubbing c. tracheal deviation d. limited chest expansion e. increased tactile fremitus f. u se of accessory muscles

d. limited chest expansion e. increased tactile fremitus

6. A severely obese patient has undergone Roux-en-Y gastric bypass surgery. In planningpostoperative care, the nurse anticipates that the patient a. may have severe diarrhea early in the postoperative period. b. will not be allowed to ambulate for 1 to 2 days postoperatively. c. will require nasogastric suction until the incision heals. d. may have only liquids orally, and in very limited amounts, during the early postoperative period.

d. may have only liquids orally, and in very limited amounts, during the early postoperative period. Roux-en-Y: only a small part of the stomach is used to create a new stomach pouch, roughly the size of an egg. The smaller stomach is connected directly to the middle portion of the small intestine (jejunum), bypassing the rest of the stomach and the upper portion of the small intestine (duodenum).

A patient's ABGs include a PaO2 of 88mg Hg, PaCO2 of 38 mm Hg, mixed venous blood gases include PvO2 40 mm Hg, and PvCO2 of 46 mm Hg. What do these findings indicate? a. impaired cardiac output b. unstable hemodynamics c. inadequate delivery of oxygen to the tissues d. normal capillary oxygen-carbon dioxide exchange

d. normal capillary oxygen-carbon dioxide exchange

5. A patient with a stable, closed fracture of the humerus caused by trauma to the arm has a temporary splint with bulky padding applied with an elastic bandage. The nurse suspects compartment syndrome and notifies the physician when the patient experiences a. increasing edema of the limb. b. muscle spasms of the lower arm. c. rebounding pulse at the fracture site. d. pain when passively extending the fingers.

d. pain when passively extending the fingers.

1. A patient is admitted to the hospital with a diagnosis of diarrhea with dehydration. The nurse recognizes that increased peristalsis resulting in diarrhea can be related to a. sympathetic inhibition. b. mixing and propulsion. c. sympathetic stimulation. d. parasympathetic stimulation.

d. parasympathetic stimulation.

4. The nurse's responsibility for a patient with a suspected disc herniation who is experiencing acute pain and muscle spasms is a. encouraging total bed rest for several days. b. teaching the principles of back strengthening exercises. c. stressing the importance of straight-leg raises to decrease pain. d. promoting the use of cold and hot compresses and pain medication.

d. promoting the use of cold and hot compresses and pain medication.

(CH 41) What is a postoperative nursing intervention for the obese patient who has undergone bariatric surgery a. irrigation and re positioning the nasogastric tube as needed b. delaying ambulation until the patient has enough strength to support self c. keeping the patient positioned on the side to facilitate respiratory function d. providing adequate support to the incision during coughing, deep breathing, and turning

d. providing adequate support to the incision during coughing, deep breathing, and turning

3. A complete nutritional assessment including anthropometric measurements is important for the patient who a. has a BMI of 25.5 kg/m2. b. complains of frequent nocturia. c. reports a 5-year history of constipation. d. reports an unintentional weight loss of 10 lb in 2 months.

d. reports an unintentional weight loss of 10 lb in 2 months.

5. A patient has the following arterial blood gas results: pH 7.52; PaCO2 30 mm Hg; HCO3 − 24 mEq/L. The nurse determines that these results indicate a. metabolic acidosis. b. metabolic alkalosis. c. respiratory acidosis. d. respiratory alkalosis

d. respiratory alkalosis Normal pH: 7.35-7.45 Normal PaCO2: 35-45 mmHg Notmal HCO3: 21-28 mEq/L

(CH 23) When obtaining important health information from a patient during assessment of the skin, it is important for the nurse to ask about a. a history of freckles as a child b. patterns of weight gain and loss. c. communicable childhood illnesses d. skin problems related to the use of medications

d. skin problems related to the use of medications

1. During the postoperative care of a 76-year-old patient, the nurse monitors the patient's intake and output carefully, knowing that the patient is at risk for fluid and electrolyte imbalances primarily because a. older adults have an impaired thirst mechanism and need reminding to drink fluids. b. water accounts for a greater percentage of body weight in the older adult than in younger adults. c. older adults are more likely than younger adults to lose extracellular fluid during surgical procedures. d. small losses of fluid are more significant because body fluids account for only about 50% of body weight in older adults.

d. small losses of fluid are more significant because body fluids account for only about 50% of body weight in older adults.

1. Sleep is best described as a a. loosely organized state similar to coma. b. state in which pain sensitivity decreases. c. quiet state in which there is little brain activity. d. state in which an individual lacks conscious awareness of the environment.

d. state in which an individual lacks conscious awareness of the environment.

5. In teaching a patient with SLE about the disorder, the nurse knows that the pathophysiology of SLE includes a. circulating immune complexes formed from IgG autoantibodies reacting with IgG. b. an autoimmune T-cell reaction that results in destruction of the deep dermal skin layer. c. immunologic dysfunction leading to chronic inflammation in the cartilage and muscles. d. the production of a variety of autoantibodies directed against components of the cell nucleus.

d. the production of a variety of autoantibodies directed against components of the cell nucleus.

1. The nurse suspects an ankle sprain when a patient at the urgent care center relates a. being hit by another soccer player during a game. b. having ankle pain after sprinting around the track. c. dropping a 10-lb weight on his lower leg at the health club. d. twisting his ankle while running bases during a baseball game.

d. twisting his ankle while running bases during a baseball game.

An older patient is describing increased ridigity in the shoulders, back and hips. The loss of elasticity in what tissue contributes to this? a: actin b: Fascia c: Myosin d: Cartilage

d: Cartilage

The nurse identifies the presence of age associated memory impairment in the older adult who states: a: I just can't seem to remember the name of my granddaughter b: I make out lists to help me remember what I need to do, but I can't seem to use them. c: I forgot that I went to the grocery store this morning and didn't realize it until I went again this afternoon d: I forget movie stars' names more often now but I can remember them later after the conversation is over.

d: I forget movie stars' names more often now but I can remember them later after the conversation is over.

Which medication is an amphetamine wake-promotion drug? a: Modafinil (Provigil) b: Protriptyline (Vavactil) c: Desipramine (Norpramin) d: Methylphenidate (Concerta)

d: Methylphenidate (Concerta)

What is a typical parasomnia? a: Cataplexy b: Hypopnea c: Sleep Apnea d: Sleep Terrors

d: Sleep Terrors

An older patient asks the nurse why she has so much trouble sleeping. What is the most appropriate response by the nurse? a: Disturbed sleep is a normal part of aging b: have you tried any over-the-counter medications to help you sleep? c: don't worry. You don't need as much sleep as you did when you were younger d: tell me more about the trouble you are having. There may be some things we can do to help.

d: Tell me more about the trouble you are having. There may be some things we can do to help.

Following change-of-shift handoff, which patient should the nurse assess first? a: a 58 YO male experiencing phantom pain and requesting analgesic b: a 72 YO male being transferred to a skilled nursing unit following repair of a hip fracture c: a 25 YO female in the left leg skeletal traction asking for the weights to be lifted for a few minutes d: a 68 YO male with a new lower leg cast complaining that the cast is too tight and he cannot feel his toes

d: a 68 YO male with a new lower leg cast complaining that the cast is too tight and he cannot feel his toes

A postoperative 68 Y.O. opioid-naive patient is receiving morphine by patient controlled analgesia (PCA) for postoperative pain. What is the rationale for not initiating the PCA analgesic with a basal dose of analgesic as well? a: opioid overdose b: nausea and itching c: lack of pain control d: adverse respiratory outcomes

d: adverse respiratory outcomes

While caring for a patient following up uvulopalatopharyngoplasty (UPPP), the nurse monitors the patient for which complications in the immediate postoperative period? a: snoring and foul-smelling breathe b: infection and electrolyte imbalance c: loss of voice and severe sore throat d: airway obstruction and hemorrhage

d: airway obstruction and hemorrhage

A 65 YO patient has undergone a right total hip arthoplasty with a cemented prosthesis for treatment of severe osteoarthritis of the hip. What is included in the activity the nurse anticipates for the patient on the patient's first or second postoperative day. a: transfer from the bed to the chair twice a day only b: turning from the back to the unaffected side q2hr only c: crutch walking with non-weight bearing on the operative leg d: ambulation and weight bearing on the right leg with a walker

d: ambulation and weight bearing on the right leg with a walker

A young patient with a fracture femur has a hip spica cast applied. While the cast is drying, what should the nurse do? a: elevate the legs about the level of the heart for 24 hours b: turn the patient to both sides and prone to supine every 2 hours c: cover the cast with a light blanket to avoid chilling from evaporation d: assess the patient frequently for abdominal pain, nausea and vomiting.

d: assess the patient frequently for abdominal pain, nausea and vomiting.

The patient works on a computer 8 hours each day. What kind of repetitive strain injury would be expected in this patient? a: meniscus injury b: rotator cuff injury c: radial-ulnar fracture d: carpal tunnel syndrome

d: carpal tunnel syndrome

A 72 Y.O. man tells the nurse that he cannot perform most of the physical activities he could do 5 years ago because for the overall joint aches and pains. What can the nurse do to assist the patient to prevent further deconditioning and decrease the risk for developing musculoskeletal problems? a: limit weight-bearing exercise to prevent stress on fragile bones and possible hip fractures b: advise the patient to avoid the use of canes and walkers because the increase dependence on ambulation aids c: advise the patient to increase his activity by more frequently climbing stairs in buildings and other environments with steps d: discuss the use of OTC medications to decrease inflammation and pain so that exercise can be maintained

d: discuss the use of OTC medications to decrease inflammation and pain so that exercise can be maintained

What test provides fast, precise measurement of the bone mass of the spine, forearm, and total body to evaluate? a: bone scan b: diskogram c: quantitative ultrasound (QUS) d: dual energy x-ray absorptiometry (DXA)

d: dual energy x-ray absorptiometry (DXA)

The patient is diagnosed with torticollis. What should the nurse be prepared to provide for the patients. a: an immobilizer to hold the bones in place b: exercises to increase the strength of the muscles c: a pillow to use to support the knees while sleeping d: enough pillows to support the patient's head carefully

d: enough pillows to support the patient's head carefully Torticollis: A rare condition in which the neck muscles contract, causing the head to twist to one side.

What is the best description of the periosteum? a: lining of a joint capsule b: a characteristic of skeletal muscle c: most common type of cartilage tissue d: fibrous connective tissue covering bone

d: fibrous connective tissue covering bone

What should the nurse include in discharge instructions for the patient following a hip prosthesis with a posterior approach? a: restrict walking for 2-3 months b: take a bath rather than a shower to prevent falling c: keep the leg internally rotated while sitting and standing d: have a family member put on the patient's shoes and socks.

d: have a family member put on the patient's shoes and socks.

When working with older patients who identify with a specific ethnic group, the nurse recognizes that health care problems may occur in these patients because they: a: live with extended families who isolate the patient b: live in rural areas where services are not readily available c: eat ethnic foods that do not provide all essential nutrients d: have less income to spend for medications and health care services

d: have less income to spend for medications and health care services

A 78-year-old woman has physiologic change related to aging in her joints. What is an appropriate nursing intervention related to common changes of aging in the musculoskeletal system? a: encourage rest to eliminate fatigue b: provide all care for the patient to ensure that care is completed c: encourage eating enough calories to avoid the risk for impaired skin integrity. d: have the patient exercise to maintain muscle strength and avoid the risk for falls

d: have the patient exercise to maintain muscle strength and avoid the risk for falls

According to the Corbin and Strauss chronic illness trajectory, which statement describes a patient with an unstable condition? a: life-threatening situation b: increasing disability and symptoms c: gradual return to an acceptable way of life d: loss of control over symptoms and disease course

d: loss of control over symptoms and disease course

Priority Decision: A patient has fallen in the bathroom of the hospital room and reports pain in the upper right arm and elbow. Before splinting the injury, the nurse knows that the priority management of a possible fracture should include which action? a: elevation of the arm b: application of ice to the site c: notification of the health care provider d: neurovascular checks below the site of the injury

d: neurovascular checks below the site of the injury

An older adult woman is admitted to the emergency department after falling at home. The nurse cautions her not to put weight on the leg after finding what in the patient assessment? a: inability to move the toes and ankle b: edema from the thigh extending to the knee c: internal rotation of the leg with groin pain d: shortening and external rotation of the leg

d: shortening and external rotation of the leg

When assessing the patient the nurse notices that the patient has a footdrop and the foot slaps down on the floor as the patient walks. How does the nurse document this gait? a: ataxic gait b: spastic gait c: antalgic gait d: steppage gait

d: steppage gait

While having his height measured during a routine health examination, a 79 Y.O. man asks the nurse why he is "shrinking." How should the nurse explain the decreased height that occurs with aging. a: decreased muscle mass results in a stooped posture b: loss of cartilage in the knees and hip joints causes a loss of height c: long bones become less dense and shorten as bone tissue compacts d: vertebrae become more compressed with thinning of intervertebral discs.

d: vertebrae become more compressed with thinning of intervertebral discs.

Which age-related changes in the respiratory system caused decreased secretion clearance? (select all) a. decreased functional cilia b. decreased force of cough c. decreased chest wall compliance d. small airway closure earlier in expiration e. decreased functional immunoglobulin IgA

decreased functional cilia decreased force of cough

Which electrolyte imbalance is associated with the following cause? (may be one or more) Prolonged immobilization __________ a. Hypernatremia b. Hyponatremia c. Hyperkalemia d. Hypokalemia e. Hypercalcemia f. Hypocalcemia g. Hyperphosphatemia h. Hypophosphatemia i. Hypermagnesemia j. Hypomagnesemia

e. Hypercalcemia

Which electrolyte imbalance is associated with the following cause? (may be one or more) Parathyroidectomy ________ a. Hypernatremia b. Hyponatremia c. Hyperkalemia d. Hypokalemia e. Hypercalcemia f. Hypocalcemia g. Hyperphosphatemia h. Hypophosphatemia i. Hypermagnesemia j. Hypomagnesemia

f. Hypocalcemia

Which electrolyte imbalance is associated with the following cause? (may be one or more) Vitamin D deficiency __________ a. Hypernatremia b. Hyponatremia c. Hyperkalemia d. Hypokalemia e. Hypercalcemia f. Hypocalcemia g. Hyperphosphatemia h. Hypophosphatemia i. Hypermagnesemia j. Hypomagnesemia

f. Hypocalcemia

Which electrolyte imbalance is associated with the following cause? (may be one or more) Chronic alcoholism ______________ a. Hypernatremia b. Hyponatremia c. Hyperkalemia d. Hypokalemia e. Hypercalcemia f. Hypocalcemia g. Hyperphosphatemia h. Hypophosphatemia i. Hypermagnesemia j. Hypomagnesemia

f. Hypocalcemia j. Hypomagnesemia

Which electrolyte imbalance is associated with the following cause? (may be one or more) Fleet enemas __________ a. Hypernatremia b. Hyponatremia c. Hyperkalemia d. Hypokalemia e. Hypercalcemia f. Hypocalcemia g. Hyperphosphatemia h. Hypophosphatemia i. Hypermagnesemia j. Hypomagnesemia

g. Hyperphosphatemia

Which electrolyte imbalance is associated with the following cause? (may be one or more) Alcohol withdrawal _________ a. Hypernatremia b. Hyponatremia c. Hyperkalemia d. Hypokalemia e. Hypercalcemia f. Hypocalcemia g. Hyperphosphatemia h. Hypophosphatemia i. Hypermagnesemia j. Hypomagnesemia

h. Hypophosphatemia

Which electrolyte imbalance is associated with the following cause? (may be one or more) Milk of Magnesia use in renal failure __________ a. Hypernatremia b. Hyponatremia c. Hyperkalemia d. Hypokalemia e. Hypercalcemia f. Hypocalcemia g. Hyperphosphatemia h. Hypophosphatemia i. Hypermagnesemia j. Hypomagnesemia

i. Hypermagnesemia

What is Felty Syndrome?

most common in patients with severe nodule forming RA (nodlues occur in 20%-30% of RA patients). It's characterized by splenomegaly and leukopenia

which respiratory defense mechanism is most impaired by smoking? a. cough reflex b. mucociliary clearance c. filtration of air d. reflex bronchoconstriction

mucociliary clearance Ciliary action impaired by smoking and increased mucus production may be caused by the irritants in tobacco smoke, leading to impairment of the mucociliary clearance system

The abnormal assessment findings of dullness and hyperresonance are found with which assessment technique? a. inspection b. palpation c. percussion d. auscultation

percussion

What keeps alveoli from collapsing?

surfactant


Related study sets

Your Money and Credit Exam 2 Cengage Questions

View Set

What Was the Great Chicago Fire questions

View Set

Chapter 7: Participation And Voting

View Set

Final - Community assessment, Community Change, Family assessment

View Set

OB PrepU: Difficulty Conceiving a Child: Chapter 8

View Set

2) Humans, Reason, and Animals (ch 8)

View Set

EMT - Chapter 17: Neurologic Emergencies

View Set